You are on page 1of 158

Precalculus

Activity Sheets

These Activity Sheets were collaboratively developed and


reviewed by educators from public schools. We encourage
teachers and other education stakeholders to email their
feedback, comments and recommendations to the Department
of Education at action _____________
We value your feedback and recommendations.

Department of Education
Republic of the Philippines

i
Precalculus
Activity Sheets

Republic Act 8293. Section 176 states that: No copyright shall subsist in any work
of the Government of the Philippines. However, prior approval of the government agency or
office wherein the work is created shall be necessary for exploitation of such work for profit.
Such agency or office may, among other things, impose as a condition the payment of royalties.

Borrowed materials (i.e., activities, pictures, photos, brand names, trademarks, etc.)
included in this activity sheets are owned by their respective copyright holders. DepEd is
represented by the _______________________________________________________ is
seeking permission to use these activity sheets from their respective copyright owners. All
means have been exhausted in seeking permission to use these activity sheets.

Published by the ____________________


__________________________________

Development Team of the Precalculus Activity Sheets


Chester C. Ambat Raquel M. Palma
Kimberly C. Ventura
Quality Assurance Team
Violeta L. Meneses Jocelyn DG. Estacio
Agnes A. Abrigo Conrado C. Carpio, Jr.
Alma Rebecca L. De Guzman Belinda A. Lalas
Wilma S. Carrera, Ed.D.
Carmina C. Gutierrez, Ed.D.
Sheila Marie A. Primicias, Ed.D
Management Team of the Precalculus Activity Sheets
DepEd, SDO 1 Pangasinan
DepEd Region I

ii
To the Precalculus Learners

The Precalculus course bridges Basic Mathematics and Calculus. This course completes the
foundational knowledge on Algebra, Geometry, and Trigonometry of students who are
planning to take courses in the STEM track. It provides them with conceptual understanding
and computational skills that are crucial for Basic Calculus and future STEM courses.
The development of the Activity Sheets (AS) was based on the Most Essential Learning
Competencies (MELC) issued by the DepEd Central Office. The Activity Sheets shall be
utilized by learners during home learning sessions.
The primary aim of these Activity Sheets is to help our learners in time of crisis happening
today. Let’s join hand-in-hand so that education will continue for every Filipino Learner.
At the end of each activity, the learners will be able to solve other precalculus problems and
reinforce ideas and skills. Each Activity Sheet contains exercises with four levels of difficulty
(Simple, Moderate, Challenging and Enrichment).
Answers in the Activity Sheets are provided at the end of every quarter to guide you while
solving them. We hope that you will use this to grow more in studying Precalculus and help
you in the near future. We encourage you to carefully study each activity with the guidance of
your teacher and parents. Although great effort has been put into these Activity Sheets for
technical correctness and precision, any mistake found and reported to the Team is a gain for
other learners. Thank you for your cooperation.

The Precalculus AS Team

iii
Table of Contents
Quarter 1, Week 1
Name the Conic ……………………………………………………………………1
Circle Me …………………………………………………………………………..2
Graph Me a Circle …………………………………………………………………3
Graph Me a Circle …………………………………………………………………6
Quarter 1, Week 2
Find My Vertex …………………………………………………………………....8
Graph Me a Parabola ……………………………………………………………....9
Graph Me a Parabola ………………………………………………………………11
Graph Me a Parabola ………………………………………………………………15
Quarter 1, Week 3
Find My Center …………………………………………………………………….17
The Graph of Ellipse ……………………………………………………………….18
The Graph of Ellipse ……………………………………………………………….21
The Graph of Ellipse ……………………………………………………………….23
Quarter 1, Week 4
Where am I? ………………………………………………………………………..25
Draw Me a Hyperbola ……………………………………………………………...26
Draw Me a Hyperbola ……………………………………………………………...28
Draw Me a Hyperbola ……………………………………………………………...30
Quarter 1, Week 5
Identify Me ………………………………………………………………………….33
Identify Me ………………………………………………………………………….34
Identify Me ………………………………………………………………………….35
Identify Me ………………………………………………………………………….37
Quarter 1, Week 6 – 7
Solve Me in Three Little Ways! …………………………………………………….39
Solve and Graph Me Up …………………………………………………………….41
Solve Me Basically! ………………………………………………………………...45

iv
SLE on the Go! ……………………………………………………………………..47
Quarter 1, Week 8 – 9
Triple S Activity …………………………………………………………………....48
SeqSer and Sigma Notation on its Evaluation ……………………………………...50
Triple S Application ………………………………………………………………...52
Wrapping Up Activity! …….……………………………………………………......53
Answer Key ...........................................................................................................................54
Quarter 2, Week 1 – 2
Round and Round Activity ………………………………………………………….76
Draw and Compute Me Logically! ………………………………………………….79
App-App Activity ……………………………………………………………...…....81
Wrapped Me in Round Activity! ……………………………………………………83
Quarter 2, Week 3
Tracking The Starting Point ………………………………………………………...84
Tracking The Starting Point ………………………………………………………...86
Tracking The Starting Point ………………………………………………………...88
Tracking The Starting Point ………………………………………………………...90
Quarter 2, Week 4
Draw Me Meticulously ……………………………………………………………...92
Draw Me Meticulously ……………………………………………………………...94
Draw Me Meticulously ……………………………………………………………...96
Draw Me Meticulously ……………………………………………………………...98
Quarter 2, Week 5
Problems of What Goes Around ……………………………………………………100
Problems of What Goes Around ……………………………………………………101
Problems of What Goes Around ……………………………………………………102
Problems of What Goes Around ……………………………………………………103
Quarter 2, Week 6
The Use of Identity to Find it”s Value ………………………………………………104
The Use of Identity to Find it”s Value ………………………………………………106

v
The Use of Identity to Find it”s Value ………………………………………………108
The Use of Identity to Find it”s Value ………………………………………………110
Quarter 2, Week 7
The Use of Identity to Solve It’s Problem …………………………………………..112
The Use of Identity to Solve It’s Problem …………………………………………..113
The Use of Identity to Solve It’s Problem …………………………………………..115
The Use of Identity to Solve It’s Problem …………………………………………..116
Quarter 2, Week 8
Limited Yet Existing Values ………………………………………………………..118
Limited Yet Existing Values ………………………………………………………..119
Limited Yet Existing Values ………………………………………………………..121
Limited Yet Existing Values ………………………………………………………..123
Quarter 2, Week 9
Problems of the Opposite World ……………………………………………………125
Problems of the Opposite World ……………………………………………………126
Problems of the Opposite World ……………………………………………………127
Problems of the Opposite World ……………………………………………………129
Answer Key ..........................................................................................................................131
References ………………………………………………………………………………….149

vi
Name: __________________________________________________ Date: _____________
Grade/Section: ___________________________________________ Score: ____________

Title of the Activity: Name the Conic


Most Essential Learning Competency: Illustrate the different types of Conic Sections,
define a circle and graph a circle given an equation in center-radius form.
K to 12 BEC CG: STEM_PC11AG-Ia-1, STEM_PC11AG-Ia-2 and
STEM_PC11AG-Ia-4

Directions: Answer the following activity accurately.

I. Identify the conic sections given in the following statements:

________1. This is a type of conic section formed when the plane (tilted) intersects only one
cone to form a bounded curve.

________2. This is a type of conic section formed when the plane (not necessarily vertical)
intersects both cones to form two unbounded curves.

________3. This is a type of conic section formed when the plane intersects only one cone to
form an unbounded curve.

________4. This a type of conic section formed when the plane intersects the cone
horizontally.

1
Name: __________________________________________________ Date: _____________
Grade/Section: ___________________________________________ Score: ____________

Title of the Activity: Circle me


Most Essential Learning Competency: Illustrate the different types of Conic Sections,
define a circle and graph a circle given an equation in center-radius form.
K to 12 BEC CG: STEM_PC11AG-Ia-1, STEM_PC11AG-Ia-2 and
STEM_PC11AG-Ia-4

Directions: Answer the following activities accurately.

I. Identify the center and radius of the following circles.

1. (𝑥 − 2)2 + (𝑦 + 3)2 = 4
Center: ________ Radius: ________
2. (𝑥 − 3)2 + (𝑦 + 1)2 = 16
Center: ________ Radius: ________
3. (𝑥 − 1)2 + (𝑦 + 6)2 = 9
Center: ________ Radius: ________
4. 𝑥 2 + (𝑦 − 4)2 = 14
Center: ________ Radius: ________
5. (𝑥 − 2)2 + 𝑦 2 = 6
Center: ________ Radius: ________

II. Identify the center and radius of the following circles in general form.
1. 𝑦 2 + 4𝑥 − 16 − 2𝑦 = −𝑥 2
Center: ________ Radius: ________
2. −𝑦 2 − 𝑥 2 = −9
Center: ________ Radius: ________
3. 2𝑦 − 𝑦 2 − 6𝑥 − 𝑥 2 = 9
Center: ________ Radius: ________
4. 𝑥 2 + 𝑦 2 − 8𝑥 − 6𝑦 = −20
Center: ________ Radius: ________
5. 𝑥 2 + 𝑦 2 + 6𝑥 − 2𝑦 + 9 = 0
Center: ________ Radius: ________

2
Name: _____________________________________________ Date: _____________
Grade/Section: _______________________________________ Score:_____________

Title of the Activity: Graph Me a Circle


Most Essential Learning Competency: Illustrate the different types of Conic Sections,
define a circle and graph a circle given an equation in center-radius form.
K to 12 BEC CG: STEM_PC11AG-Ia-1, STEM_PC11AG-Ia-2 and
STEM_PC11AG-Ia-4

Directions: Answer the following activities accurately.

I. Determine the standard form equation, center and radius of the following circles.
Sketch the circles.
1. 8𝑥 + 𝑥 2 − 2𝑦 = 60 − 𝑦 2
Standard Equation: __________________________

Center: ________
Radius: _______

2. 𝑥 2 + 𝑦 2 + 14𝑥 − 12𝑦 + 8 = 0
Standard Equation: __________________________
Center: ________
Radius: ________

3
3. 𝑦 2 + 2𝑥 + 𝑥 2 = 2𝑦 − 120
Standard Equation: __________________________
Center: ________
Radius: ________

4. 𝑥 2 + 2𝑥 + 𝑦 2 = 60 + 10𝑦
Standard Equation: __________________________
Center: ________
Radius: ________

5. 8𝑥 + 32𝑦 + 𝑦 2 = −270 − 𝑥 2
Standard Equation: __________________________
Center: ________
Radius: ________

4
II. Use the information to identify the standard equation form of the following circles.
Sketch the circles.

1. Center (-8, -16), radius √8 2. Center (-10, -5), radius 4

3. (𝑥 − 18)2 + (𝑦 − 8)2 = 1
Translated 4 left, 2 up

4. (𝑥 + 6)2 + (𝑦 − 8)2 = 1
Translated 4 left, 4 down

5
Name: ______________________________________________ Date: _____________
Grade/Section: ________________________________________ Score: ____________

Title of the Activity: Graph Me a Circle


Most Essential Learning Competency: Illustrate the different types of Conic Sections,
define a circle and graph a circle given an equation in center-radius form.
K to 12 BEC CG: STEM_PC11AG-Ia-1, STEM_PC11AG-Ia-2 and
STEM_PC11AG-Ia-4

Directions: Use the information to identify the standard equation form of


the following circles. Sketch the circles.

1. Center (-11, -14)


Area= 16π

Standard equation:

_____________________

2. Center (-5,12)
Circumference: 8 π

Standard equation:

_____________________

6
3. Center (2, -5)
Point on circle (-7, -1)

Standard equation:

_____________________

4. Center (14,17)
Point on Circle (15,17)

Standard equation:

_____________________

7
Name: _______________________________________________ Date: _____________
Grade/Section: ________________________________________ Score: ____________

Title of the Activity: Find My Vertex


Most Essential Learning Competency: Define a Parabola and graph a parabola given
an equation in vertex form.
K to 12 BEC CG: STEM_PC11AG-Ia-5 & STEM_PC11AG-Ib-2

Directions: Identify the vertex of the following parabola.

1. 𝑥 = (𝑦 − 5)2 − 1 Vertex: ____________________

2. 𝑥 = (𝑦 + 3)2 − 1 Vertex: ____________________

3. 𝑥 = −(𝑦 − 5)2 + 1 Vertex: ____________________

1 1
4. 𝑦 = − 2 𝑥 2 + 2 Vertex: ____________________

1
5. 𝑦 = 2 (𝑥 − 5)2 + 2 Vertex: ____________________

8
Name: __________________________________________________Date: _____________
Grade/Section:____________________________________________ Score:____________

Title of the Activity: Graph Me a Parabola


Most Essential Learning Competency: Define a parabola and graph a parabola given
an equation in vertex form.
K to 12 BEC CG: STEM_PC11AG-Ia-5 and STEM_PC11AG-Ib-2

Directions: Answer the following activities accurately.

I. Determine the vertex, focus, directrix and axis of symmetry of the following
parabola.
1. 𝑦 = −(𝑥 + 4)2 − 1
Vertex: ________ Focus: _______ Directrix: _______ Axis of Symmetry: ______

1
2. 𝑥 = − 4 (𝑦 + 5)2

Vertex: _______ Focus: _______ Directrix: _______ Axis of Symmetry: ______

1
3. − 4 (𝑥 − 3) = (𝑦 + 5)2

Vertex: _______ Focus: _______ Directrix: _______ Axis of Symmetry: ______

4. −(𝑦 + 4) = (𝑥 − 2)2
Vertex: _______ Focus: _______ Directrix: _______ Axis of Symmetry: ______

5. 𝑥 = −2(𝑦 + 6)2
Vertex: ________ Focus: _______ Directrix: _______ Axis of Symmetry:______

9
II. Identify the vertex, focus, directrix, axis of symmetry of the following parabola.
Sketch the parabola.

1. −2𝑥 2 − 4𝑥 + 𝑦 + 85 = 0

Vertex: ________ Focus: _______ Directrix:________ Axis of Symmetry:_____

2. −2𝑦 2 + 𝑥 + 20𝑦 = −65


Vertex: ________ Focus: _______ Directrix:________ Axis of Symmetry:______

10
Name: ____________________________________________ Date: _____________
Grade/Section: _____________________________________ Score:_____________

Title of the Activity: Graph Me a Parabola


Most Essential Learning Competency: Define a parabola and graph a parabola given
an equation in vertex form.
K to 12 BEC CG: STEM_PC11AG-Ia-5 and STEM_PC11AG-Ib-2

Directions: Answer the following activities accurately.

I. Utilize the information to determine the standard equation form (vertex form)
of the following parabola. Sketch the parabola.

1. Vertex (-1, -6), Focus (-17/16, -6)


Standard Equation: __________________________

11
2. Vertex (-3,3), focus (-47/16,3)
Standard Equation: __________________________

3. Vertex (-8,6), directrix: y=10/3


Standard Equation: __________________________

12
4. Vertex (-7,10), passes through (-5,9)
Standard Equation: ______________________

5. Axis of symmetry is horizontal, passes through (-5, -4),(-10,28),(-4,4).


Standard Equation: __________________________

13
II. Determine the vertex, latus rectum, length of latus rectum, x-intercept and y-
intercept of the following parabola. Sketch the parabola.

1. 26𝑥 − 50 + 𝑦 − 2𝑥 2 = 0
Vertex: __________

Latus rectum: __________

Length of latus rectum: ___________

x-intercept: __________

y-intercept: __________

2. 15𝑦 2 + 𝑥 = 210𝑦 − 702

Vertex: __________

Latus rectum: __________

Length of latus rectum: ___________

x-intercept: __________

y-intercept: __________

14
Name: __________________________________________________ Date: _____________
Grade/Section: ____________________________________________Score: _____________

Title of the Activity: Graph Me a Parabola


Most Essential Learning Competency: Define a Parabola and graph a parabola given
an equation in vertex form.
K to 12 BEC CG: STEM_PC11AG-Ia-5 and STEM_PC11AG-Ib-2

Directions: Answer the following activities accurately.

I. Determine the vertex, focus, latus rectum, axis of symmetry and the directrix of
each parabola and sketch the parabola.

1. 𝑦 = 3(𝑥 + 12)2 + 1
Vertex: __________
Focus: __________
Latus rectum: __________
Directrix: __________
Axis of symmetry: __________

1
2. 𝑦 = − 3 (𝑥 − 10)2 + 1

Vertex: __________
Focus: __________
Latus rectum: __________
Directrix: __________

15
1 16 86
3. 𝑦 = − 3 𝑥 2 + 𝑥−
3 5

Vertex: __________
Focus: __________
Latus rectum: __________
Directrix: __________
Axis of symmetry: __________

4. 𝑦 = 2𝑥 2 + 36𝑥 + 143
Vertex: __________
Focus: __________
Latus rectum: __________
Directrix: __________
Axis of symmetry: __________

5. 𝑥 = 𝑦 2 + 4𝑦 − 5
Vertex: __________
Focus: __________
Latus rectum: __________
Directrix: __________
Axis of symmetry: __________

16
Name: __________________________________________________ Date: _____________
Grade/Section :___________________________________________ Score:_____________

Title of the Activity: Find My Center


Most Essential Learning Competency: Define an ellipse and graph an ellipse given
in standard equation.
K to 12 BEC CG: STEM_PC11AG-Ic-1 and STEM_PC11AG-Ic-3

Directions: Identify the center of each ellipse.

(𝑥+4)2 (𝑦−3)2
1. + =1 Center: __________
16 4

(𝑥−4)2 (𝑦+2)2
2. 5
+ 15
=1 Center: __________

(𝑥−2)2 (𝑦+6)2
3. + =1 Center: __________
9 4

𝑥2 (𝑦−3)2
4. + =1 Center: __________
49 9

(𝑥−3)2 𝑦2
5. + 16 = 1 Center: __________
9

17
Name: _________________________________________________ Date: _____________
Grade/Section: ___________________________________________Score: ____________

Title of the Activity: The Graph of Ellipse


Most Essential Learning Competency: Define an ellipse and graph an ellipse given
in standard equation.
K to 12 BEC CG: STEM_PC11AG-Ic-1 and STEM_PC11AG-Ic-3

III. Directions: Answer the following activities accurately.

I. Determine the center, vertices, co-vertices of each of the following ellipse.


𝑥2 𝑦2
1. + 36 = 1
49

Center: __________ Vertices: ____________ Co-vertices: ____________

𝑥2 𝑦2
2. + 50 = 1
90

Center: __________ Vertices: ____________ Co-vertices: ____________

𝑥2 𝑦2
3. + 81 = 1
172

Center: __________ Vertices: ____________ Co-vertices: _____________

(𝑥−24)2 (𝑦−9)2
4. 64
+ 121
=1

Center: __________ Vertices: ____________ Co-vertices: ____________

(𝑥+5)2 (𝑦−6)2
5. + =1
144 81

Center: __________ Vertices: ____________ Co-vertices:_____________

18
II. Identify the center, vertices, co-vertices of the following ellipse. Sketch the
ellipse.

𝑥2 𝑦2
1. 9
+ 16 = 1

Center: __________ Vertices: ____________ Co-vertices:_____________

𝑥2 (𝑦−3)2
2. + =1
25 16

Center: __________ Vertices:____________ Co-vertices:_______________

19
(𝑥−8)2 𝑦2
3. 49
+ 36 = 1

Center: __________ Vertices: ______________ Co-vertices:___________

20
Name: __________________________________________________ Date: _____________
Grade/Section: ____________________________________________Score: ____________

Title of the Activity: The Graph of Ellipse


Most Essential Learning Competency: Define an ellipse and graph an ellipse given
in standard equation.
K to 12 BEC CG: STEM_PC11AG-Ic-1, and STEM_PC11AG-Ic-3

Directions: Determine the standard equation, center, vertices, co-vertices,


foci, length of major axis and length of minor axis of the following ellipse.
Sketch the ellipse.

1. −2𝑥 2 − 8𝑥 − 𝑦 2 = −16𝑦 + 70

Center: _____________________
Vertices: ____________________
Co-vertices: __________________
Foci: _______________________
Length of major axis: __________
Length of minor axis: __________

2. 4𝑦 2 + 169𝑥 2 − 338𝑥 + 32𝑦 = 536

Center: _____________________
Vertices: ____________________
Co-vertices: __________________
Foci: _______________________
Length of major axis: __________
Length of minor axis: __________

21
3. 9𝑦 2 + 4𝑥 2 − 8𝑥 + 126𝑦 = 152

Center: _____________________
Vertices: ____________________
Co-vertices: __________________
Foci: _______________________
Length of major axis: __________
Length of minor axis: __________

4. 16𝑥 2 + 25𝑦 2 − 32𝑥 + 100𝑦 − 336

Center: _____________________
Vertices: ____________________
Co-vertices: __________________
Foci: _______________________
Length of major axis: __________
Length of minor axis: __________

5. 9𝑥 2 + 4𝑦 2 − 36𝑥 + 24𝑦 + 76

Center: _____________________
Vertices: ____________________
Co-vertices: __________________
Foci: _______________________
Length of major axis: __________
Length of minor axis: __________

22
Name: __________________________________________________ Date: _____________
Grade/Section: ____________________________________________ Score: ____________

Title of the Activity: The Graph of Ellipse


Most Essential Learning Competency: Define an ellipse and graph an ellipse given
in standard equation.
K to 12 BEC CG: STEM_PC11AG-Ic-1, and STEM_PC11AG-Ic-3

Directions: Utilize the information provided to identify the standard equation


of each ellipse and sketch the ellipse.

1. Vertices (16,0), (-16,0)

Co-vertices (0,8), (0,-8)

Standard equation:

_______________________________

2. Vertices (14,0), (-14,0)

Co-vertices (7,0), (-7,0)

Standard equation:

_______________________________

23
3. Foci (6, -2), (0, -2)

Co-vertices (3,2), (3, -6)

Standard equation:

_______________________________

4. Center (10,5)

Vertex (10,13)

Focus (10,1.5)

Standard equation:

_______________________________

24
Name: __________________________________________________ Date: _____________
Grade/Section: ___________________________________________ Score: ____________

Title of the Activity: Where am I?


Most Essential Learning Competency: Define a hyperbola and graph a hyperbola
given in standard equation.
K to 12 BEC CG: STEM_PC11AG-Id-1 and STEM_PC11AG-Id-3

Directions: Identify the center, major axis and minor axis of the following
hyperbola.

𝑥2 𝑦2
1. − 25 = 1
16

Center: __________ Major Axis: __________ Minor Axis: __________

(𝑦+4)2 (𝑥−3)2
2. − =1
9 16

Center: __________ Major Axis: __________ Minor Axis: __________

𝑦2
3. − (𝑥 + 4)2 = 1
25

Center: __________ Major Axis: __________ Minor Axis: __________

𝑦2 (𝑥−1)2
4. − =1
25 16

Center: __________ Major Axis: __________ Minor Axis: __________

(𝑥−5)2 (𝑦+4)2
5. − =1
32 16

Center: __________ Major Axis: __________ Minor Axis:__________

25
Name: __________________________________________________ Date: _____________
Grade/Section: ____________________________________________Score: ____________

Title of the Activity: Draw Me a Hyperbola


Most Essential Learning Competency: Define a hyperbola and graph a hyperbola
given in standard equation.
K to 12 BEC CG: STEM_PC11AG-Id-1 and STEM_PC11AG-Id-3

Directions: Answer the following activities accurately.

I. Identify the center, vertices, length of major axis, length of minor axis of the
following hyperbola.

𝑥2 𝑦2
1. 64
− 25 = 1

Center: __________ Vertices: _______________


Length of major axis: ________ Length of minor axis: ________

𝑥2 (𝑦+2)2
2. − =1
16 9

Center: __________ Vertices: _______________


Length of major axis: ________ Length of minor axis: ________

(𝑦+4)2 (𝑥−6)2
3. − =1
9 4

Center: __________ Vertices: _______________


Length of major axis: ________ Length of minor axis: ________

26
II. Identify the center, vertices, foci and asymptotes of the following the following
hyperbola. Sketch the hyperbola.
(𝑥+2)2 (𝑦−5)2
1. − =1
144 25

Center: __________ Vertices: _____________________________


Foci: _______________________
Asymptotes: ________________________________________________

(𝑦+8)2 (𝑥−6)2
2. − =1
25 9

Center: __________ Vertices: _____________________________


Foci: _______________________
Asymptotes: ________________________________________________

27
Name: __________________________________________________ Date: _____________
Grade/Section: ____________________________________________Score: ____________

Title of the Activity: Draw Me a Hyperbola


Most Essential Learning Competency: Define a hyperbola and graph a hyperbola
given in standard equation.
K to 12 BEC CG: STEM_PC11AG-Id-1 and STEM_PC11AG-Id-3

Directions: Answer the following activities accurately.

I. Identify the standard form equation of the following hyperbola. Sketch the
hyperbola.
1. −10𝑦 − 𝑦 2 + 4𝑥 2 + 72𝑥 = −300
Standard Equation: ___________________________

2. 𝑥 2 − 18𝑥 − 𝑦 2 + 12𝑦 − 32 = 0
Standard Equation: ___________________________

28
3. 9𝑥 2 − 4𝑦 2 + 16𝑦 − 18𝑥 = 67
Standard Equation: ___________________________

II. Utilize the information provided to identify the standard form equation of each
hyperbola. Sketch the hyperbola.

1. Vertices (1,9), (13,9) Standard Equation: _______________________


Focus (-2,9)

2. Center (-6,9) Standard Equation: ________________________


Vertex (-6,15)
Length of minor axis: 12 units

29
Name: __________________________________________________ Date: _____________
Grade/Section: ___________________________________________ Score: ____________

Title of the Activity: Draw Me a Hyperbola


Most Essential Learning Competency: Define a hyperbola and graph a hyperbola
given in standard equation.
K to 12 BEC CG: STEM_PC11AG-Id-1 and STEM_PC11AG-Id-3

Directions: Answer the following activities accurately.

I. Determine the vertices, foci, asymptotes, length of transverse axis and conjugate
axis and eccentricity of each hyperbola. Sketch the hyperbola.

1. 𝑥 2 − 𝑦 2 − 6𝑥 = −16𝑦 + 230
Vertices: _______________ Length of transverse axis: ______________
Foci: __________________ Length of conjugate axis: _______________
Asymptotes: ____________ Eccentricity: _________________
______________________

30
2. −4𝑥 2 + 𝑦 2 + 8𝑥 = 20𝑦 − 10
Vertices: _______________ Length of transverse axis: ______________
Foci: __________________ Length of conjugate axis: _______________
Asymptotes: ____________ Eccentricity: _________________
______________________

3. −𝑦 2 + 𝑥 2 = −12𝑦 + 18𝑥 − 36
Vertices: _______________ Length of transverse axis: ______________
Foci: __________________ Length of conjugate axis: _______________
Asymptotes: ____________ Eccentricity: _________________
______________________

31
II. Utilize the information to identify the standard form equation of hyperbola. Sketch
the hyperbola.

1. Vertices (0, -1), (20, -1)


Asymptotes: 𝑦 = 𝑥 + 9, 𝑦 = −𝑥 − 11

Standard Equation: _____________________________________

2. Focus (1,12),
1 1
Asymptotes: 𝑦 = 3 𝑥 + 5/3, 𝑦 = − 3 𝑥 + 7/3

Standard Equation: _____________________________________

32
Name: __________________________________________________ Date: _____________
Grade/Section: ___________________________________________ Score: ____________

Title of the Activity: Identify Me


Most Essential Learning Competency: Recognize the equation and important
characteristics of the different types of conic sections and solve situational problems
involving conic sections.
K to 12 BEC CG: STEM_PC11AG-Ie-1 and STEM_PC11AG-Ie-2

Directions: Classify each conic section. Write your answers on the space
provided.

1. 5𝑥 2 − 3𝑦 2 = −10𝑥 + 12𝑦 + 46
_______________________________
2. 2𝑦 2 = 5𝑥 + 2𝑦 + 17
_______________________________
3. 3𝑥 2 + 3𝑦 2 + 42𝑥 − 12𝑦 = −202
_______________________________
4. 7𝑥 2 + 3𝑦 2 − 14𝑥 + 12𝑦 = −21
_______________________________
5. 3𝑥 2 + 6𝑥 + 4𝑦 = 36
_______________________________
6. 𝑥 2 + 𝑦 2 = 90
_______________________________
𝑥2 𝑦2
7. + =1
9 25

_______________________________
𝑦2 𝑥2
8. − =1
25 9

_______________________________
𝑦2
9. (𝑥 − 4)2 + 16 = 1

_______________________________
10. 𝑦 = (𝑥 − 10)2 + 8
_______________________________

33
Name: __________________________________________________ Date: _____________
Grade/Section: ____________________________________________Score: ____________

Title of the Activity: Identify Me


Most Essential Learning Competency: Recognize the equation and important
characteristics of the different types of conic sections and solve situational problems
involving conic sections.
K to 12 BEC CG: STEM_PC11AG-Ie-1 and STEM_PC11AG-Ie-2

Directions: Classify each conic section and give its equation in standard
form.

1. −𝑥 2 + 10𝑥 + 𝑦 − 32 = 0
Classification: ___________________________________
Standard equation: ________________________________

2. 𝑥 2 + 𝑦 2 + 6𝑥 − 2𝑦 + 18 = 0
Classification: ___________________________________
Standard equation: ________________________________

3. 𝑥 2 − 𝑦 2 − 2𝑥 − 16 = 0
Classification: ___________________________________
Standard equation: ________________________________

4. −𝑦 2 + 𝑥 + 8𝑦 − 77 = 0
Classification: ___________________________________
Standard equation: ________________________________

5. −9𝑥 2 + 𝑦 2 − 72𝑥 − 260 = 0


Classification: ___________________________________
Standard equation: ________________________________

34
Name: __________________________________________________ Date: _____________
Grade/Section: ___________________________________________Score: _____________

Title of the Activity: Identify Me


Most Essential Learning Competency: Recognize the equation and important
characteristics of the different types of conic sections and solve situational problems
involving conic sections.
K to 12 BEC CG: STEM_PC11AG-Ie-1 and STEM_PC11AG-Ie-2

Directions: Answer each problem and enclose your final answers.

1. Smile FM radio station is 3 km east and 6 km north of the center of Pangasinan.


Smile FM can only be heard clearly 5 km from the station. Provide an equation for the
boundary where the radio station can be clearly heard.

Solution:

2. A bridge in Pangasinan is a semielliptical arch shaped and have a span of 100 feet. The
height of the arch, at a distance of 40 feet from the center is to be 10 feet. Find the
height of the arch at its center.

Solution:

35
3. The cables of a suspension bridge are in the shape of a parabola. The towers supporting
the cables are 400ft apart and 100ft tall. If the supporting cable that runs from tower to
tower is only 30 feet from the road at its closest point. Find the length of one of the
vertical support cables that is 60 feet from the towers.
Solution:

4. Erick is standing at one focus of a whispering gallery, is 6 feet from the nearest wall.
His friend Clarence is standing at the other focus, 100 feet away. What is the length of
this whispering gallery? How high is its elliptical ceiling at the center?
Solution:

36
Name: __________________________________________________ Date: _____________
Grade/Section: ___________________________________________ Score: ____________

Title of the Activity: Identify Me


Most Essential Learning Competency: Recognize the equation and important
characteristics of the different types of conic sections and solve situational problems
involving conic sections.
K to 12 BEC CG: STEM_PC11AG-Ie-1 and STEM_PC11AG-Ie-2

Directions: Answer each problem and enclose your final answers.

1. A ferris wheel is elevated 1 m above ground. When a car reaches the highest point on
the ferris wheel, its altitude from ground level is 31 m. How far away from the center,
horizontally, is the car when it is at an altitude of 25 m?
Solution:

2. A satellite dish is shaped like a paraboloid, with the receiver placed at the focus. It is to
have a depth of 0.44 m at the vertex, with the receiver placed 0.11 m away from the
vertex. What should the diameter of the satellite dish be?
Solution:

37
3. A big room is constructed so that the ceiling is a dome that is semielliptical in shape. If
a person stands at one focus and speaks, the sound that is made bounces o↵ the ceiling
and gets reflected to the other focus. Thus, if two people stand at the foci (ignoring their
heights), they will be able to hear each other. If the room is 34 m long and 8 m high,
how far from the center should each of two people stand if they would like to whisper
back and forth and hear each other?
Solution:

4. Two control towers are located at points Q(500,0) and R(500,0), on a straight shore
where the x-axis runs through (all distances are in meters). At the same moment, both
towers sent a radio signal to a ship out at sea, each traveling at 300 m/µs. The ship
received the signal from Q 3 µs (microseconds) before the message from R. Find the
equation of the curve containing the possible location of the ship.
Solution:

38
Name: _______________________________________________ Date: _____________
Grade/Section: ________________________________________ Score: ____________

Title of the Activity: Solve Me in Three Little Ways!


Most Essential Learning Competency: Illustrate systems of nonlinear equations,
determine the solutions of systems of equations (one linear and one second-degree) in
two variables using substitution, elimination, and graphing (in standard form) and solve
situational problems involving systems of nonlinear equations.
K to 12 BEC CG: STEM_PC11AG-If-1, STEM_PC11AG-If-g-1
and STEM_PC11AG-Ig-2

Directions: Answer the following activities accurately.

I. Solve each system by graphing.

1
1. y = − 4 𝑥 − 4
5
y = 4𝑥 + 2

2. y = 2x + 3
y = −4𝑥 − 3

3. 7x − y = 3
x − y = −3

39
4. 3x + 4y = 4
3x + 4y = −16

II. Solve each system by substitution.

1. x + 8y = −15
7x + 8y = −9

2. −5x−7y = 11
x −2y = −9

3. y = −7x + 1
5x + 4y = −19

4. −9x−3y = −2
y = −3x−4

III. Solve each system by elimination.

1. 6x + 2y = −6
7x + 4y = 8

2. 5x + 3y = 15
10x + 6y = 20

3. −6x − 9y = 0
−24x = 36y

4. –3 – 3y = 12x
–5 – y = 2x

40
Name: _______________________________________________ Date: _____________
Grade/Section: ________________________________________ Score: ____________

Title of the Activity: Solve and Graph Me Up!


Most Essential Learning Competency: Illustrate systems of nonlinear equations,
determine the solutions of systems of equations (one linear and one second-degree) in
two variables using substitution, elimination, and graphing (in standard form) and solve
situational problems involving systems of nonlinear equations.
K to 12 BEC CG: STEM_PC11AG-If-1, STEM_PC11AG-If-g-1
and STEM_PC11AG-Ig-2

Directions: Answer the following activities accurately.

I. Use either substitution or elimination method to solve the system, and sketch the
graphs in one Cartesian plane showing the point of intersection.

1. x + y = 1
y = x2 – 5

2. x2 + y2 = 9
y=x+3

41
3. x + y = –1
x2 + y2 = 13

4. y = x + 3
xy = –2

5. (x + 2)2 + (y – 2)2 = 1
y = –(x+2)2 + 3

6. x2 + 2y2 = 11
3x2 – 2y2 = 25

42
7. 2x2 + 3y2 = 30
3x2 – 2y2 = 19

II. State if the point given is a solution to the system of equations.

1. x2 + y2 + 15x + 3y + 23 = 0
–2x + y = 0
Point: (–7, –8)

2. 4x2 + y2 – 6x + 3y – 72 = 0
2x – y = 0
Point: (–3, –6)

3. x2 + y2 – 7x + 3y – 28 = 0
–2x + y – 4 = 0
Point: (3, –5)

4. –y2 + x – 12y – 33 = 0
–y2 + x – 24y – 81= 0
Point: (–1, –4)

5. –x2 + 2y2 – 2x + 8y + 5 = 0
–x2 + 26y2 – 2x + 104y + 77 = 0
Point: (–1, –3)

43
III. Solve each system of equations.

1. 3x2 + 6y2 – 3x + 7y – 7 = 0
x+y=3

2. 5x2 – y2 – 4x – 12y – 92 = 0
x–y=2

3. x2 – y2 + 10x + 16y – 43 = 0
10x2 – y2 + 100x + 16y + 146 = 0

4. x2 + 2x – 16y – 47 = 0
x2 + 17y2 + 2x + 69y + 55 = 0

5. 2x2 + 14x + y – 10 = 0
–2x2 – 14x + y + 22 = 0

6. 5x2 + 30x + 48y – 8 = 0


5x2 + 10y2 + 30x + 78y – 48 = 0

7. 2y2 + x – 20y + 7 = 0
16y2 + x – 160y + 7 = 0

8. 5x2 + 50x – 12y + 108 = 0


5x2 + 17y2 + 50x – 148y – 45 = 0

9. x2 + y2 – 16x – 12y + 95 = 0
–x2 + 10y2 + 16x + 78y + 37 = 0

10. x2 + y2 + 12x – 10y – 60 = 0


x2 + y2 – 2x – 10y + 10 = 0

44
Name: _______________________________________________ Date: _____________
Grade/Section: ________________________________________ Score: ____________

Title of the Activity: Solve Me Basically!


Most Essential Learning Competency: Illustrate systems of nonlinear equations,
determine the solutions of systems of equations (one linear and one second-degree) in
two variables using substitution, elimination, and graphing (in standard form) and solve
situational problems involving systems of nonlinear equations.
K to 12 BEC CG: STEM_PC11AG-If-1, STEM_PC11AG-If-g-1
and STEM_PC11AG-Ig-2

Directions: Solve the following word problems involving nonlinear equations,

1. Find the value of two numbers if their sum is 12 and their difference is 4.

2. The difference of two numbers is 3. Their sum is 13. Find the numbers.

3. Flying to Kampala with a tailwind a plane averaged 158 km/hr. On the return trip the
plane only averaged 112 km/hr while flying back into the same wind. Find the speed of
the wind and the speed of the plane in still air.

4. The sum of the digits of a certain two-digit number is 7. Reversing its digits increases
the number by 9. What is the number?

5. A boat traveled 210 miles downstream and back. The trip downstream took 10 hours.
The trip back took 70 hours. What is the speed of the boat in still water? What is the
speed of the current?

6. The state fair is a popular field trip destination. This year the senior class at High School
A and the senior class at High School B both planned trips there. The senior class at
High School A rented and filled 8 vans and 8 buses with 240 students. High School B
rented and filled 4 vans and 1 bus with 54 students. Every van had the same number of
students in it as did the buses. Find the number of students in each van and in each bus.

45
7. The senior classes at High School A and High School B planned separate trips to New
York City. The senior class at High School A rented and filled 1 van and 6 buses with
372 students. High School B rented and filled 4 vans and 12 buses with 780 students.
Each van and each bus carried the same number of students. How many students can a
van carry? How many students can a bus carry?

8. Matt and Ming are selling fruit for a school fundraiser. Customers can buy small boxes
of oranges and large boxes of oranges. Matt sold 3 small boxes of oranges and 14 large
boxes of oranges for a total of Php203. Ming sold 11 small boxes of oranges and 11
large boxes of oranges for a total of Php220. Find the cost each of one small box of
oranges and one large box of oranges.

9. A boat traveled 336 miles downstream and back. The trip downstream took 12 hours.
The trip back took 14 hours. What is the speed of the boat in still water? What is the
speed of the current?

10. At Elisa's Printing Company LLC there are two kinds of printing presses: Model A
which can print 70 books per day and Model B which can print 55 books per day. The
company owns 14 total printing presses and this allows them to print 905 books per
day. How many of each type of press do they have?

46
Name: _______________________________________________ Date: _____________
Grade/Section: ________________________________________ Score: ____________

Title of the Activity: SLE on the Go!


Most Essential Learning Competency: Illustrate systems of nonlinear equations,
determine the solutions of systems of equations (one linear and one second-degree) in
two variables using substitution, elimination, and graphing (in standard form) and solve
situational problems involving systems of nonlinear equations.
K to 12 BEC CG: STEM_PC11AG-If-1, STEM_PC11AG-If-g-1
and STEM_PC11AG-Ig-2

Directions: Solve the following situational problem applying the systems of


nonlinear equations.

1. A laptop has screen size 13 inches with aspect ratio 5 : 4. Find the length and
the width of the screen. Answer: 10.15 in ⇥ 8.12 in

2. What are the dimensions of a rectangle whose perimeter is 50 cm and diagonal 18 cm?

3. Lacy is speeding in her car, and sees a parked police car on the side of the road right
next to her at t=0 seconds. She immediately decelerates, but the police car accelerates
to catch up with her. (Assume that two cars are going in the same direction in parallel
paths).
The distance that Lacy has travelled in feet after t seconds can be modeled by
the equation d (t) = 150 + 75t – 1.2t2. The distance that the police car travels after
t seconds can be modeled by the equation d (t) = 4t2.
a. How long will it take the police car to catch up to Lacy?
b. How many feet has Lacy travelled from the time she saw the police car (time
t=0) until the police car catches up to Lacy?

47
Name: ______________________________________________ Date: _____________
Grade/Section: _______________________________________ Score: ____________

Title of the Activity: Triple S Activity


Most Essential Learning Competency: Illustrate series, differentiate a series from a
sequence, use the sigma notation to represent a series, and apply the use of sigma
notation in finding sums
K to 12 BEC CG: STEM_PC11SMI-Ih-1, STEM_PC11SMI-Ih-2
and STEM_PC11SMI-Ih-3

Directions: Answer the following activities accurately.

I. Write SEQ if the given item is a sequence, and write SER if it is a series.
1. 1, 2, 4, 8, . . .
2. 2, 8, 10, 18, . . .
3. −1 + 1 − 1 + 1 − 1
1 2 3 4
4. , , ,
2 3 4 5

5. 1 + 2 + 22 + 23 + 24
6. 1 + 0.1 + 0.001 + 0.0001

II. Write A if the sequence is arithmetic, G if it is geometric, F if Fibonacci, and O if


it is not one of the mentioned types.
1. 3, 5, 7, 9, 11, . . .
2. 2, 4, 9, 16, 25, . . .
1 1 1 1
3. , , , ,...
4 16 64 256
1 2 3 4
4. , ,
3 9 27 81
, ,...
1 1 1 1 1
5. , , , , ,...
5 9 13 17 21

6. 4, 6, 10, 16, 26, . . .


7. √3, √4, √5, √6, . . .
8. 0.1, 0.01, 0.001, 0.0001, . . .

48
III. Identify the series (and write NAGIG if it is not arithmetic, geometric, and infinite
geometric series), and determine the sum (and write NO SUM if it cannot be
summed up).
1. 4 + 9 + 14 + · · · + 64

1
2. 81 + 27 + 9 + · · · + 81

3. 1 + 3 + 6 + 10 + 15 + 21 + · · · + 55

4. −10 − 2 + 6+· · · + 46

5. 10 + 2 + 0.4 + 0.08 + · · ·

1 1 1 1
6. + + + +. . .
2 3 5 7

7. 1 − 0.1 + 0.01 − 0.001 + · · ·

IV. Expand each summation, and simplify if possible.

1. ∑5k=−1(2 − 3k)

2. ∑nj=1 x j

3. ∑6j=3 j2 − j

4. ∑4k=1(−1)k+1 k

5. ∑3n=1 an + 1 − an

49
Name: ______________________________________________ Date: _____________
Grade/Section: _______________________________________ Score: ____________

Title of the Activity: SeqSer and Sigma Notation on its Evaluation


Most Essential Learning Competency: Illustrate series, differentiate a series from a
sequence, use the sigma notation to represent a series, and apply the use of sigma
notation in finding sums
K to 12 BEC CG: STEM_PC11SMI-Ih-1, STEM_PC11SMI-Ih-2
and STEM_PC11SMI-Ih-3

Directions: Answer the following activities accurately.

I. Rewrite each series as a sum.


1. ∑4𝑛=1(20 − 𝑛2 )
2. ∑4𝑛=1(𝑛 + 600)
1
3. ∑4𝑚=1 3𝑚
𝑎2 +1
4. ∑6𝑎=1 𝑎

II. Evaluate each series.


1. ∑11
m=5(m + 400)
10
2. ∑5n=1 n
300
3. ∑6k=1 k

4. ∑5n=0(20 − n)
5. ∑5k=1(200 − k2 )
6. ∑8n=2 n
1
7. ∑9n=3 n
a
8. ∑7a=1 a+1

50
III. Evaluate each arithmetic and geometric series described.
1. ∑35
𝑘=1(5𝑘 − 2)

2. ∑35
𝑖=1(3𝑖 − 13)

3. ∑15
𝑚=1 4𝑚

4. ∑10
𝑚=1(7𝑚 − 2)

5. ∑6𝑖=1 3𝑖
6. ∑45
𝑛=1(3𝑛 − 9)

7. ∑8𝑘=1 −2 ∙ 6𝑘−1
1 𝑚−1
8. ∑8𝑚=1 32 ∙ (2)

9. ∑10
𝑖=1 0.2 ∙ 5
𝑖−1

10. ∑10
𝑛=1 −2 ∙ 2
𝑛−1

IV. Rewrite each series using sigma notation.


1. 1 + 4 + 9 + 16 + 25
1 1 1 1
2. 1 + 2 + 3 + 4 + 5
1 1 1 1 1 1
3. + 9 + 27 + 81 + 243 + 729
3

4. 5 + 10 + 15 + 20 + 25
1 2 3 4 5
5. +3+4+5+6
2

6. 5 + 25 + 125 + 625 + 3125

51
Name: ______________________________________________ Date: _____________
Grade/Section: _______________________________________ Score: ____________

Title of the Activity: Triple S Application


Most Essential Learning Competency: Illustrate series, differentiate a series from a
sequence, use the sigma notation to represent a series, and apply the use of sigma
notation in finding sums
K to 12 BEC CG: STEM_PC11SMI-Ih-1, STEM_PC11SMI-Ih-2
and STEM_PC11SMI-Ih-3

Directions: Apply the use of sigma notation in finding sums.

I.
1. ∑50
𝑘=1(2 − 3𝑘)

2. ∑𝑛𝑗=1(1 + 2𝑗)

1
3. ∑99
𝑗=1 √𝑖+1+√1

II.
3𝑔(𝑖)−𝑓(𝑖)+7
1. If ∑30 30 30
𝑖=1 𝑓 (𝑖 ) = 70 and ∑𝑖=1 𝑔(𝑖 ) = 50, what is the value of ∑𝑖=1 ?
2

2. If ∑𝑛𝑖=1(𝑖 + 1)2 = an3 + bn2 + cn + d, what is a + b + c + d?

3. If s = ∑100 200
𝑖=1 𝑖 , express ∑𝑖=1 𝑖 in terms of s.

52
Name: ______________________________________________ Date: _____________
Grade/Section: _______________________________________ Score: ____________

Title of the Activity: Wrapping Up Activity!


Most Essential Learning Competency: Illustrate series, differentiate a series from a
sequence, use the sigma notation to represent a series, and apply the use of sigma
notation in finding sums
K to 12 BEC CG: STEM_PC11SMI-Ih-1, STEM_PC11SMI-Ih-2
and STEM_PC11SMI-Ih-3

Directions: Solve the following problem.

1. If s = ∑𝑛𝑖=1 𝑎1 , does it follow that ∑𝑛𝑖=1 𝑎𝑖2 = 𝑠 2 ? Why or why not?

2. Are these equal? Why or why not?


1 1
∑50 70
𝑥=1 and ∑𝑥=21
𝑥 𝑥−20

3. Rewrite the following so that it starts at x = 0


∑10
𝑥=7 𝑥(𝑥 + 1)

4. Derive a formula for ∑𝑛𝑖=1 𝑖 3 by using a telescoping sum with terms f (i) = i4.

53
Answer Key:
Simple Activities: Name the Conic
1. Ellipse
2. Hyperbola
3. Parabola
4. Circle
Moderate Activities: Circle Me
1. Center (2, -3), radius: r=2
2. Center (3, -1), radius: r=4
3. Center (1, -6), radius: r=3
4. Center (0,4), radius: r= √14
5. Center (2,0), radius: r= √6
6. Center (-2,1), radius: r=√21
7. Center (0,0), radius: r=3
8. Center (-3,1), radius: r=√3
9. Center (4,3), radius: r=√5
10. Center (-3,1), radius: r=1

Challenging Activities: Graph Me a Circle


I.
1. Standard equation: (𝑥 + 4)2 + (𝑦 − 1)2 = 77
Center (-4,1)
Radius: r=√77
Graph:

2. Standard equation: (𝑥 + 4)2 + (𝑦 − 1)2 = 77


Center (-7,6)
Radius: r=√77
Graph:

54
3. Standard equation: (𝑥 + 1)2 + (𝑦 − 12)2 = 25
Center (-1,12)
Radius: r= 5
Graph:

4. Standard equation: (𝑥 + 1)2 + (𝑦 − 5)2 = 86


Center (-1,5)
Radius: r= √86
Graph:

5. Standard equation: (𝑥 + 4)2 + (𝑦 + 16)2 = 2


Center (-1,5)
Radius: r= √86
Graph:

II.
1. Standard equation: (𝑥 + 10)2 + (𝑦 + 5)2 = 16
2. Standard equation: (𝑥 + 8)2 + (𝑦 + 16)2 = 8
3. Standard equation: (𝑥 − 14)2 + (𝑦 − 10)2 = 8
4. Standard equation: (𝑥 + 10)2 + (𝑦 + 12)2 = 25

55
Enrichment Activities: Graph Me a Circle
1. Standard equation: (𝑥 + 11)2 + (𝑦 + 14)2 = 16
Graph:

2. Standard equation: (𝑥 + 5)2 + (𝑦 − 12)2 = 16


Graph:

3. Standard equation: (𝑥 − 2)2 + (𝑦 + 5)2 = 97


Graph:

4. Standard equation: (𝑥 − 14)2 + (𝑦 − 17)2 = 1


Graph

56
Answer Key:
Simple Activities: Find My Vertex
1. Vertex (-1,5)
2. Vertex (-1, -3)
3. Vertex (1,5)
4. Vertex (0,1/2)
5. Vertex (5,2)

Moderate Activities: Graph Me a Parabola


I 1. Vertex (-4, -1)
Focus (-4, -5/4)
Directrix: X=-3/4
Axis of symmetry: X=-4

2. Vertex (0, -5)


Focus (-1, -5)
Directrix: X= 1
Axis of symmetry: Y=-5

3. Vertex (3, -5)


Focus (47/16, -5)
Directrix: X=49/16
Axis of symmetry: Y= -5

4. Vertex (2, -4)


Focus (2, -17/4)
Directrix: X=-15/4
Axis of symmetry: X= 2

5. Vertex (0, -6)


Focus (-1/8, -6)
Directrix: X= 1/8
Axis of symmetry: Y= -6

57
II. 1. Vertex (-1,87)
Focus (-1, -695/8)
Directrix: Y=-695/8
Axis of symmetry: X= -1

a. Vertex (-15, -5)


Focus (-121/8, -5)
Directrix: X=-119/8
Axis of symmetry: Y= -5

Challenging Activities: Graph Me a Parabola


I.
1. Standard equation: 𝑥 = −4(𝑦 + 6)2 − 1

2. Standard equation: 𝑥 = 4(𝑦 + 3)2 − 3

3
3. Standard equation: 𝑦 = 32 (𝑥 + 8)2 + 6

58
1
4. Standard equation: 𝑦 = 10 − 4 (𝑥 + 7)2

3 16 2 191
5. Standard equation: 𝑥 = − 256 (𝑦 − ) −
3 48

II.
1. Vertex (13/2, -69/2)
Latus rectum y=-277/8
Length of latus rectum: ½ unit
x-intercept: (2.35,0), (10.65,0)
y-intercept: (0,50)

59
2. Vertex (33,7)
Latus rectum y= 1981/60
Length of latus rectum: 1/15 unit
x-intercept: (-702,0)
y-intercept: (0,5.52), (0, 8.48)

Enrichment Activities: Graph Me a Parabola


1. Vertex (-12,1)
Focus (-12,13/12)
Latus rectum y=13/12
Directrix: y = 11/12
Axis of symmetry: x= -12

2. Vertex (10,1)
Focus (10,1/4)
Latus rectum y=1/4
Directrix: y = 7/4
Axis of symmetry: x= 10

60
3. Vertex (8,62/15)
Focus (8, 203/60)
Latus rectum y = 203/60
Directrix: y = 293/60
Axis of symmetry: x= 8

4. Vertex (-9, -19)


Focus (-9, -151/8)
Latus rectum y= -151/8
Directrix: y = -153/8
Axis of symmetry: x= -9

5. Vertex (-9,-2)
Focus (-35/4, -2)
Latus rectum x = -35/4
Directrix: x = -37/4
Axis of symmetry: y = -2

61
Answer Key:
Simple Activities: Find My Center
1. Center (-4,3)
2. Center (4,2)
3. Center (2, -6)
4. Center (0,3)
5. Center (3,0)

Moderate Activities: The Graph of Ellipse


I.
1. Center (0,0)
Vertices (-7,0), (7,0)
Co-vertices (0,6), (0, -6)

2. Center (0,0)
Vertices (-3 √10,0), (3 √10,0)
Co-vertices (0, -5 √2), (2 √10,0)

3. Center (0,0)
Vertices (-2 √43,0), (2 √43,0)
Co-vertices (0, -9), (0,9)

4. Center (24,9)
Vertices (24, -2), (24,20)
Co-vertices (16,9), (32,9)

5. Center (-5,6)
Vertices (-17,6), (7,6)
Co-vertices (-5, -3), (-5,15)

62
II.
1. Center (0,0)
Vertices (0, -4), (0,4)
Co-vertices (-3,0), (3,0)

2. Center (0,3)
Vertices (-5,3), (5,3)
Co-vertices (0, -1), (0,7)

3. Center (8,0)
Vertices (1,0), (15,0)
Co-vertices (8, -6), (8,7)

Challenging Activities: The Graph of Ellipse


(𝑦−8)2
1. Standard equation: (𝑥 + 2)2 + 2
=1

Center (-2,8)
Vertices (-2,8-√2), (-2,8+√2)
Co-vertices (-3,8), (-1,8)
Foci: (-2,7), (-2,9)
Length of major axis: 2 √2 units
Length of minor axis: 2 units

63
63 2
676(𝑥−1)2 16(𝑦+ )
4
2. Standard equation: + =1
5253 5253

Center (1, -63/4)


Vertices (1, -33.87), (1,2.37)
Co-vertices (-1.79, -15.75), (3.79, -15.75)
Foci: (1, -33.65), (1, 2.15)
Length of major axis: 36.24 units
Length of minor axis: 5.58 units

49(𝑥−1)2 3(𝑦+7)2
3. Standard equation: + =1
579 199

Center (1, -7)


Vertices (-11.22, -7), (13.22, -7)
Co-vertices (1, -15.14), (1,1.14)
Foci: (-8.11, -7), (10.11, -7)
Length of major axis: 12.22 units
Length of minor axis: 16.29 units

4(𝑥−1)2 25(𝑦+2)2
4. Standard equation: + =1
113 452

Center (1, -2)


Vertices (-4.32, -2), (6.32, -2)
Co-vertices (1, -6.25), (1,2.25)
Foci: (-2.19, -2), (4.19, -2)
Length of major axis: 10.63 units
Length of minor axis: 8.5 units

9(𝑥−2)2 4(𝑦+3)2
5. Standard equation: + =1
49 49

Center (2, -3)


Vertices (2, -6.5), (2,0.5)
Co-vertices (-0.33, -3), (4.33, -3)
Foci: (2, -0.39), (2, -5.61)
Length of major axis: 7 units
Length of minor axis: 4.67 units

64
Enrichment Activities: The Graph of Ellipse

𝑥2 𝑦2
1. Standard equation: 256 + 64 = 1

𝑥2 𝑦2
2. Standard equation: 196 + 147 = 1

(𝑥−3)2 (𝑦+2)2
3. Standard equation: + =1
25 16

4(𝑥−10)2 (𝑦−5)2
4. Standard equation: + =1
207 64

65
Answer Key:
Simple Activities: Where am I?
1. Center (0,0)
Major axis: x-axis
Minor axis: y-axis
2. Center (3,4)
Major axis: y-axis
Minor axis: x-axis
3. Center (-4,0)
Major axis: y-axis
Minor axis: x-axis
4. Center (1,0)
Major axis: y-axis
Minor axis: x-axis
5. Center (5, -4)
Major axis: x-axis
Minor axis: y-axis

Moderate Activities: Draw Me a Hyperbola


I.
1. Center (0,0)
Vertices (-8,0), (8,0)
Length of major axis: 16 units
Length of minor axis: 10 units
2. Center (0, -2)
Vertices (-4, -2), (4,-2)
Length of major axis: 8 units
Length of minor axis: 6 units
3. Center (6, -4)
Vertices (6, -7), (6, -1)
Length of major axis: 6 units
Length of minor axis: 4 units

66
II.
1. Center (-2,5)
Vertices (-14,5), (10,5)
Foci (-15,5), (11,5)
25 5𝑥 5𝑥 35
Asymptotes: 𝑦 = 6 − 12 ; 𝑦 = 12 + 6

2. Center (6, -8)


Vertices (6, -13), (6,-3)
Foci (6, -13.83), (6, -2.17)
5𝑥 5𝑥
Asymptotes: 𝑦 = 2 − 3 ; 𝑦 = 3 − 18

Challenging Activities: Draw Me a Hyperbola


I.
1. −4(𝑥 + 9)2 + (𝑦 + 5)2 = 1

(𝑥−9)2 (𝑦−6)2
2. − =1
77 77

67
3(𝑥−1)2 (𝑦−2)2
3. − =1
20 15

II.
(𝑥−7)2 (𝑦−9)2
1. − =1
36 45

(𝑦−9)2 (𝑥+6)2
2. − =1
25 36

Enrichment Activities: Draw Me a Hyperbola


I.
1. Vertices (-10.23,8), (16.23,8)
Foci (-15.71,8), (21.71,8)
Asymptotes: 𝑦 = 11 − 𝑥 ; 𝑦 = 𝑥 + 5
Length of transverse axis: 26.46 units
Length of conjugate axis: 26.46 units
Eccentricity: √2

68
2. Vertices (1, 0.73), (1, 19.27)
Foci (1, -0.37), (1,20.37)
Asymptotes: 𝑦 = 12 − 2𝑥 ; 𝑦 = 2𝑥 + 8
Length of transverse axis: 18.55 units
Length of conjugate axis: 9.27 units
Eccentricity: 1.12

3. Vertices (6,6), (12,6)


Foci (4.76,6), (13.24,6)
Asymptotes: 𝑦 = 15 − 𝑥 ; 𝑦 = 𝑥 − 3
Length of transverse axis: 6 units
Length of conjugate axis: 6- units
Eccentricity: 1.41

II.
(𝑥+10)2 (𝑦+1)2
1. − =1
100 100

(𝑦−2)2 (𝑥−1)2
2. − =1
10 90

69
Answer Key:
Simple Activities: Identify Me
1. Hyperbola
2. Parabola
3. Circle
4. Ellipse
5. Parabola
6. Circle
7. Ellipse
8. Hyperbola
9. Ellipse
10. Parabola
Moderate Activities: Identify Me
I.
1. Classification: Parabola
Standard Equation: 𝑦 = (𝑥 − 5)2 + 7
2. Classification: Circle
Standard Equation: (𝑥 + 3)2 + (𝑦 − 1)2 = 1
3. Classification: Hyperbola
(𝑥−1)2 𝑦2
Standard Equation: − 17 = 1
17
4. Classification: Parabola
Standard Equation: 𝑥 = (𝑦 − 4)2 + 61
5. Classification: Hyperbola
𝑦2 9(𝑥+4)2
Standard Equation: 116 − =1
116

Challenging Activities: Identify Me 𝑥2 𝑦2


4. − −1
202500 47500
1.(𝑥 − 3)2 + (𝑦 − 6)2 = 52
2. Height at the center: 16.7 ft
3. 64.3 ft
4. Length 112 ft, height at the center: 25.2 ft
Enrichment Activities
1. 12 m
2. 0.88 m
3. 15 m

70
Answer Key:
Simple Activities: Solve in Three Little Ways
I. 1. II. 1. (1, −2)
2. (−5,2)
3. (1, −6)
4. No solution

2. ` III. 1. (−4,9)
2. No solution
3. Infinite number of
solution
4. (2, −9)

3. 4.

71
Moderate Activities: Solve and Graph Me Up
I.
1. 2.

3. 4.

5. 6.

72
7.

II. 1. No III. 1. No solution


2. Yes 2. (–3, –5), (6,4)
3. No 3. (–7,8), (–3,8)
4. No 4. (–1, –3), (3, –2), (–5, –2)
5. Yes 5. (–8, –6), (1, –6)
6. (–2,1), (–4,1), (4, –4), (–6, –1)
7. (–7,10), (–7,0)
8. (0,9), (–10,9),(–4, –1), (–6, –1)
9. No solution
10. (5,5)
Challenging Activities: Solve Me Basically
I. 1. 4 and 8
2. 5 and 8
3. plane: 135km/hr; wind: 23km/hr
4. 34
5. boat: 12mph; current: 9mph
6. Van: 8; bus: 22
7. Van: 18; bus: 59
8. Small and big box of oranges respectively: Php7, Php13
9. Boat: 26mph; current: 2mph
10. 9 of Model A and 5 of Model B

Enrichment Activities: SLE on the Go!


1. 10.15 in x 8.12 in 3. a. t = 16.2 seconds
2. 14.9cm x 10.1cm b. 1050 feet

73
Answer Key:
Simple Activities: Triple S Activities
I. 1. SEQ II. 1. A
2. SEQ 2. O
3. SER 3. G
4. SEQ 4. O
5. SER 5. A
6. SER 6. F
7. O
8. G
III. 1. Arithmetic, 442 IV. 1. −28
9841
2. Geometric, 81
2. x + x2 + x3 + . . . + xn

3. NAGIG, 220 3. 68
4. Aritmetic, 144 4. −2
5. Infinite Geometric, 12.5 5. 𝑎4 − 𝑎1
6. NAGIG, NO SUM
10
7. Infinite Geometric, 11

Moderate Activities: SeqSer and Sigma Notation on its Evaluation


I. 1. 19 + 16 + 11 + 4
2. 601 + 602 + 603 + 604
1 1 1 1
3. 3 + 9 + 27 + 81
5 10 17 26 37
4. 2 + 2 + 3
+ 4
+ 5
+ 6

II. 1. 2 856
137
2.
6

3. 735
4. 105
5. 945
6. 35
3349
7. 2520
1479
8. 280

74
III. 1. 3 080 IV. 1. ∑5𝑚=1 𝑚2
2. 1 435 1
2. ∑5𝑘=1 𝑘
3. 480 1
3. ∑6𝑘=1 3𝑘
4. 365
5. 63 4. ∑5𝑛=1 5𝑛
𝑎
6. 2 700 5. ∑5𝑎=1 𝑎+1

7. −671 846 6. ∑5𝑎=1 5𝑎


255
8. 4

9. 488 281.2
10. −2 046

Challenging Activities: Triple S Application


I. 1. −3 725
2. 2n + n2
3. 9

II. 1. 145
2. 4
3. 2s + 100 000

Enrichment Activities: Wrapping Up Activities!


1. No. If s = ∑2𝑖=1 𝑎1 = 𝑎1 + 𝑎2 , then ∑𝑛𝑖=1 𝑎𝑖2 = 𝑎12 + 𝑎22 , while s2 = 𝑎12 + 2a1a2 + 𝑎22 .
1 1 1
2. Yes. Both are 1 + + + + . . . to 50 terms.
2 3 4

3. ∑3𝑥=0(𝑥 + 7)(𝑥 + 8)
𝑛2 (𝑛+1)2
4. 4

75
Name: _______________________________________________ Date: _____________
Grade/Section: ________________________________________ Score: ____________

Title of the Activity: Round and Round Activity


Most Essential Learning Competency: Illustrate unit circle and the relationship
between the linear and angular measures of arcs in a unit circle, convert degree measure
to radian measure and vice versa and illustrate angles in standard position and
coterminal angles.
K to 12 BEC CG: STEM_PC11T-IIa-1, STEM_PC11T-IIa-2
and STEM_PC11T-IIa-3

Directions: Answer the following activities accurately.

I. Find the measure of each angle.


1. 6.

2. 7.

3. 8.

76
4. 9.

5. 10.

77
II. Give the degree and radian measure of the following special angles.

III. Do what is being asked.


A. Convert each degree measure into radians.
1. 240° 2. 315° 3. 125° 4. −340°

B. Convert each radian measure into degrees.


19𝜋 𝜋 43𝜋 31𝜋
1. 2. 4 3. − 4. −
4 18 18

78
Name: _______________________________________________ Date: _____________
Grade/Section: ________________________________________ Score: ____________

Title of the Activity: Draw and Compute Me Logically!


Most Essential Learning Competency: Illustrate unit circle and the relationship
between the linear and angular measures of arcs in a unit circle, convert degree measure
to radian measure and vice versa and illustrate angles in standard position and
coterminal angles.
K to 12 BEC CG: STEM_PC11T-IIa-1, STEM_PC11T-IIa-2
and STEM_PC11T-IIa-3

Directions: Answer the following activities accurately.

I. Draw an angle with the given measure in standard position.

1. 280° 6. 440°

5𝜋
2. 710° 7. 4

47𝜋
3. −120° 8. − 18

79
11𝜋
4. 9. 170°
6

10𝜋
5. − 10. 510°
3

II. Convert each decimal degree measure into degrees-minutes-seconds and each
degrees-minutes-seconds into decimal degrees.
1. 128.77° 5. 63.75°
2. 232° 7’ 57” 6. 200.325°
3. −154° 47’ 42” 7. −317.06°
4. −0.9225° 8. 179.999°

III. Do what is being asked.


A. State the quadrant in which the terminal side of each angle lies.
5𝜋
1. −509° 2. − 6

B. Find the reference angle.


47𝜋 9𝜋
1. − 3. − 5. 290°
12 4
5𝜋
2. − 4. 530° 6. −140°
4

C. Analyze the following statements.


𝜋
1. Find five angles that have a reference angle of 6 .

2. How many solutions are there to the equation x° = x radians?

80
Name: _______________________________________________ Date: _____________
Grade/Section: ________________________________________ Score: ____________

Title of the Activity: App-App Activity


Most Essential Learning Competency: Illustrate unit circle and the relationship
between the linear and angular measures of arcs in a unit circle, convert degree measure
to radian measure and vice versa and illustrate angles in standard position and
coterminal angles.
K to 12 BEC CG: STEM_PC11T-IIa-1, STEM_PC11T-IIa-2
and STEM_PC11T-IIa-3

Directions: Answer the following activities accurately.

I. Graph the oriented angle in standard position. Classify each angle according to
where its terminal side lies and then give two coterminal angles, one of which is
positive and the other negative.
11𝜋
1. 330° 4. − 3
𝜋
2. −135° 5. − 2
5𝜋
3. 6. 3𝜋
6

II. Solve the following problems.


1. A yo-yo which is 2.25 inches in diameter spins at a rate of 4500 revolutions per minute.
How fast is the edge of the yo-yo spinning in miles per hour? Round your answer to
two decimal places.

2. How many revolutions per minute would the yo-yo in Number 1 have to complete if
the edge of the yo-yo is to be spinning at a rate of 42 miles per hour? Round your answer
to two decimal places.

3. In the yo-yo trick `Around the World,' the performer throws the yo-yo so it sweeps out
a vertical circle whose radius is the yo-yo string. If the yo-yo string is 28 inches long
and the yo-yo takes 3 seconds to complete one revolution of the circle, compute the
speed of the yo-yo in miles per hour. Round your answer to two decimal places.

81
4. A computer hard drive contains a circular disk with diameter 2.5 inches and spins at a
rate of 7200 RPM (revolutions per minute). Find the linear speed of a point on the edge
of the disk in miles per hour.

82
Name: _______________________________________________ Date: _____________
Grade/Section: ________________________________________ Score: ____________

Title of the Activity: Wrapped Me in Round Activity!


Most Essential Learning Competency: Illustrate unit circle and the relationship
between the linear and angular measures of arcs in a unit circle, convert degree measure
to radian measure and vice versa and illustrate angles in standard position and
coterminal angles.
K to 12 BEC CG: STEM_PC11T-IIa-1, STEM_PC11T-IIa-2
and STEM_PC11T-IIa-3

Directions: Solve the following problems.

1. A rock got stuck in the tread of my tire and when I was driving 70 miles per hour,
the rock came loose and hit the inside of the wheel well of the car. How fast, in
miles per hour, was the rock traveling when it c9 ame out of the tread? (The tire has
a diameter of 23 inches.)

2. The Giant Wheel at Cedar Point is a circle with diameter 128 feet which sits on an
8-foot tall platform making its overall height is 136 feet. It completes two
revolutions in 2 minutes and 7 seconds. Assuming the riders are at the edge of the
circle, how fast are they traveling in miles per hour?

83
Name: _______________________________________________ Date: _____________
Grade/Section: ______________________________ Score: ____________

Title of the Activity: Tracking The Starting Point


Most Essential Learning Competency: Illustrate the different circular functions and
use reference angles to find exact values of circular functions.
K to 12 BEC CG: STEM_PC11T-IIb-1 and STEM_PC11T-IIb-2

Directions: Answer the following activities accurately

I. Give the reference angle of the following.

1. 𝜃 = 158° 6. 𝜃 = −225°

2. 𝜃 = 312° 7. 𝜃 = 420°

3. 𝜃 = 117° 8. 𝜃 = −25°

4. 𝜃 = 236° 9. 𝜃 = 1280°

5. 𝜃 = −120° 10. 𝜃 = 150

84
II. Use reference angle and appropriate sign to find the exact value.

1. sin 600°

6. sin 510 °

2. sec 585°

7. sec 60°

3. cos(−420°)

8. tan −225°

7𝜋
4. sin 6
5𝜋
9. cos 3

2𝜋
5. sec 3
3𝜋
10. tan 4

85
Name: _______________________________________________ Date: _____________
Grade/Section: ______________________________ Score: ____________

Title of the Activity: Tracking The Starting Point


Most Essential Learning Competency: Illustrate the different circular functions and
use reference angles to find exact values of circular functions.
K to 12 BEC CG: STEM_PC11T-IIb-1 and STEM_PC11T-IIb-2

Directions: Answer the following activities accurately.

I. Use reference angle and appropriate sign to find the exact value.

𝜋 4𝜋
1. sec 6 7. sin − 3

5𝜋 23𝜋
2. csc 8. sec
6 6

2𝜋 13𝜋
3. tan 9. csc
3 3

11𝜋 5𝜋
4. tan − 10. tan
6 6

35𝜋
5. cot 6

4𝜋
6. cos − 3

86
II. Find the exact value of each expression.

1. 𝑠𝑖𝑛2 150° + 𝑐𝑜𝑠 2 150°

2. cos(−30°) + sin 420°

3. tan(−225°) + tan 405°

4. sec 750° − csc(−300°)

11𝜋 5𝜋
5. sin + cos
6 3

87
Name: _______________________________________________ Date: _____________
Grade/Section: ______________________________ Score: ____________

Title of the Activity: Tracking The Starting Point


Most Essential Learning Competency: Illustrate the different circular functions and
use reference angles to find exact values of circular functions.
K to 12 BEC CG: STEM_PC11T-IIb-1 and STEM_PC11T-IIb-2

Directions: Answer the following activities accurately.

I. Find the exact value of each expression.

2𝜋 2𝜋
1. 𝑐𝑜𝑠 2 + 𝑠𝑖𝑛2
3 3

5𝜋 5𝜋
2. 2𝑐𝑜𝑠 − 𝑠𝑖𝑛
3 2

𝜋 8𝜋 13𝜋
3. 𝑡𝑎𝑛2 4 + 2𝑐𝑜𝑠 3
− 𝑠𝑖𝑛 6

2𝜋 5𝜋
𝑡𝑎𝑛 −𝑡𝑎𝑛
3 6
4. 2𝜋 5𝜋
1+𝑡𝑎𝑛 3 𝑡𝑎𝑛 6

11𝜋 𝜋
𝑠𝑖𝑛 6 −𝑐𝑜𝑠 6
5. 𝜋 5𝜋
𝑠𝑖𝑛(− 6 )+𝑐𝑜𝑠 6

88
II. Compute P(θ), and find the exact values of the six circular functions.

4𝜋
1. 𝜃 = − 3

3𝜋
2. 𝜃 = 4

III. Use the given information to find the values of the other circular functions.

1 3𝜋
1. cos 𝜃 = 𝑎𝑛𝑑 < 𝜃 < 2𝜋
2 2

8 𝜋
2. sin 𝜃 = 17 𝑎𝑛𝑑 0 < 𝜃 < 2

2√13 3𝜋
3. cos 𝜃 = 𝑎𝑛𝑑 < 𝜃 < 2𝜋
13 2

89
Name: _______________________________________________ Date: _____________
Grade/Section: ______________________________ Score: ____________

Title of the Activity: Tracking The Starting Point


Most Essential Learning Competency: Illustrate the different circular functions and
use reference angles to find exact values of circular functions.
K to 12 BEC CG: STEM_PC11T-IIb-1 and STEM_PC11T-IIb-2

Directions: Answer the following activities accurately.

I. Use the given information to find the exact values of the remaining circular
functions of θ.

3
1. sin 𝜃 = with θ in Quadrant II
5

12
2. tan 𝜃 = with θ in Quadrant III
5

25
3. csc 𝜃 = with θ in Quadrant I
24

4. sec 𝜃 = 7 with θ in Quadrant IV

10√91
5. csc 𝜃 = with θ in Quadrant III
91

6. cot 𝜃 = −23 with θ in Quadrant II

90
7. tan 𝜃 = −2 with θ in Quadrant IV

8. sec 𝜃 = −4 with θ in Quadrant II

9. cot 𝜃 = √5 with θ in Quadrant III

1
10. cos 𝜃 = with θ in Quadrant I
3

II. Compute P(θ), and find the exact values of the six circular functions.

19𝜋
1. 𝜃 = 6

32𝜋
2. 𝜃 = 3

91
Name: _______________________________________________ Date: _____________
Grade/Section: ______________________________ Score: ____________

Title of the Activity: Draw Me Meticulously


Most Essential Learning Competency: Illustrate the domain and range of the different
circular functions and graph the six circular functions (a) amplitude, (b) period, and (c)
phase shift.
K to 12 BEC CG: STEM_PC11T-IIc-1 and STEM_PC11T-IIc-d-1

Directions: Answer the following activities accurately.

I. Indicate the amplitude, period, phase shift, domain, and range for each function.

1. 𝑦 = 4 sin 𝑥

2. 𝑦 = 3 cos 𝑥

𝑥
3. 𝑦 = cos 4

4. 𝑦 = −2 sin 𝑥

5. 𝑦 = 2 + sin 4𝑥

6. 𝑦 = −1 + cos 𝑥

1
7. 𝑦 = − 2 sin 3𝑥

92
8. 𝑦 = 3𝑠𝑖𝑛(−𝑥 )

𝑥
9. 𝑦 = 3 − 2𝑐𝑜𝑠 2

𝜋
10. 𝑦 = sin (𝑥 − 4 )

III. Assuming that there is no vertical shift, find a function that describes a simple
harmonic motion with the following properties.

1. sine function; displacement zero at a time t=0; moving up initially; amplitude = 6 cm;
period = 4 sec

2. cosine function; highest point 4 cm above the equilibrium at time t = 0; period = 10


sec

3. cosine function; lowest point 9 cm below the equilibrium at time t = 0; period = 5 sec

93
Name: _______________________________________________ Date: _____________
Grade/Section: ______________________________ Score: ____________

Title of the Activity: Draw Me Meticulously


Most Essential Learning Competency: Illustrate the domain and range of the different
circular functions and graph the six circular functions (a) amplitude, (b) period, and (c)
phase shift.
K to 12 BEC CG: STEM_PC11T-IIc-1 and STEM_PC11T-IIc-d-1

Directions: Answer the following activities accurately.

I. Indicate the period, phase shift, domain, and range of each function.

1. 𝑦 = csc(−𝑥 )

2. 𝑦 = cot(−𝑥 )

3. 𝑦 = − tan 𝑥

4. 𝑦 = − sec 𝑥

5. 𝑦 = − sec 3𝑥

6. 𝑦 = 3 csc 𝑥

2𝑥
7. 𝑦 = 4 sec 3

8. 𝑦 = tan(𝑥 + 𝜋)

94
𝜋
9. 𝑦 = tan (𝑥 − 2 )

𝜋
10. 𝑦 = cot (𝑥 + )
4

II. Sketch the graph of the following functions.

1. 𝑦 = |sin 𝑥 |

𝜋
2. 𝑦 = 2 sin (2 − 𝑥) − 2

95
Name: _______________________________________________ Date: _____________
Grade/Section: ______________________________ Score: ____________

Title of the Activity: Draw Me Meticulously


Most Essential Learning Competency: Illustrate the domain and range of the different
circular functions and graph the six circular functions (a) amplitude, (b) period, and (c)
phase shift.
K to 12 BEC CG: STEM_PC11T-IIc-1 and STEM_PC11T-IIc-d-1

Directions: Answer the following activities accurately.

I. Find the transformations required to obtain the graph starting with a basic
trigonometric functions.

5𝜋
1. 𝑦 = 5 cos (𝜃 + )
6

3𝜋
2. 𝑦 = 5 + sin (8𝜃 + )
4

𝜋
3. 𝑦 = sin (𝜃 − ) − 2
6

1 𝜋
4. 𝑦 = 10 tan (𝜃 − 3 )

96
II. Find the amplitude, the period in radians, the phase shift in radians, the vertical
shift, and the minimum and maximum values. Then sketch the graph using radians.

5𝜋
1. 𝑦 = 3 sin (𝜃 − )
6

2. 𝑦 = 3 cos 𝜃

97
Name: _______________________________________________ Date: _____________
Grade/Section: ______________________________ Score: ____________

Title of the Activity: Draw Me Meticulously


Most Essential Learning Competency: Illustrate the domain and range of the different
circular functions and graph the six circular functions (a) amplitude, (b) period, and (c)
phase shift.
K to 12 BEC CG: STEM_PC11T-IIc-1 and STEM_PC11T-IIc-d-1

Directions: Answer the following activities accurately.

I. Find the period in radians, the phase shift in radians, the vertical shift, and two
vertical asymptotes (if any). Then sketch the graph using radians.

11𝜋
1. 𝑦 = tan (2𝜃 − )−1
6

𝜃 5𝜋
2. 𝑦 = 2 cot ( + )
3 6

1 𝜃 3𝜋
3. 𝑦 = csc ( + )
2 2 4

𝜃 3𝜋
4. 𝑦 = 3 sec ( − )
2 4

98
II. Find the amplitude, the period in radians, the phase shift in radians, the vertical shift,
and the minimum and maximum values. Then sketch the graph using radians.

𝜋
3. 𝑦 = 2 sin (−3𝜃 − 2 ) + 2

𝜃 𝜋
4. 𝑦 = cos ( + ) − 2
4 4

99
Name: _______________________________________________ Date: _____________
Grade/Section: ______________________________ Score: ____________

Title of the Activity: Problems of What Goes Around


Most Essential Learning Competency: Solve situational problems involving circular
functions.
K to 12 BEC CG: STEM_PC11T-IId-2

Directions: Answer the following accurately.

1. For what values of k do y = cotx and y = cot(x−kπ) have the same graph?

2. For what values of k do y = secx and y = sec(x−kπ) have the same graph?

3. Find the least positive value of c such that the graph of y = −2sin2(x + c) coincide with
that of y = 2cos2x.

4. Find the largest positive value of c such that the graph of y = 2cos3(x−c) coincide with
that of y = −2cos3(x−2).

𝜋
5. For what values of a do the graphs of y = acosb(x−c) and y = −2sec 6(x−6)never
6

intersect for any values of b and c

100
Name: _______________________________________________ Date: _____________
Grade/Section: ______________________________ Score: ____________

Title of the Activity: Problems of What Goes Around


Most Essential Learning Competency: Solve situational problems involving circular
functions.
K to 12 BEC CG: STEM_PC11T-IId-2

Directions: Answer the following accurately.

1. A point P moving in a simple harmonic motion makes 10 complete revolutions every 1


second. The amplitude of the motion is 3 m. Assuming that P is at its minimum displacement
with respect to the equilibrium when t = 0 and there is a vertical shift of 2 m downward, find a
sine function that describes the path traced by P in terms of time t.

2. The path of a fast-moving particle (assuming constant speed) traces a circle with equation
(x−3)2 + (y−4)2 = 25.
It starts at point (3,−1), moves counterclockwise, and passes the point (8,4) for the first time
after traveling 7 microseconds. Where is the particle after traveling for 20 microseconds?
Hint. The coordinates (x,y) of the location of the particle at time t (in microseconds) are given
by
𝜋 𝜋
x = 5cos 14(t−7) + 3 and y = 5sin 14(t−7) + 4.
14 14

101
Name: _______________________________________________ Date: _____________
Grade/Section: ______________________________ Score: ____________

Title of the Activity: Problems of What Goes Around


Most Essential Learning Competency: Solve situational problems involving circular
functions.
K to 12 BEC CG: STEM_PC11T-IId-2

Directions: Find the exact value of each expression.

1. A weight is suspended from a spring and is moving up and down in a simple harmonic
motion. At start, the weight is pushed up 6 cm above the restingposition, and then
released. After 14 seconds, the weight reaches again to its highest position. Find the
equation of the motion, and locate the weight with respect to the resting position after
20 seconds since it was released.

2. Suppose the lowest point of a Ferris wheel is 1.5 meters off the ground, and its radius
is 15 m. It makes one complete revolution every 30 seconds. Starting at the lowest point,
find a cosine function that gives the height above the ground of a riding child in terms
of the time t in seconds.

102
Name: _______________________________________________ Date: _____________
Grade/Section: ______________________________ Score: ____________

Title of the Activity: Problems of What Goes Around


Most Essential Learning Competency: Solve situational problems involving circular
functions.
K to 12 BEC CG: STEM_PC11T-IId-2

Directions: Use the given information to find the exact values of the remaining
circular functions of θ.

1. A wooden ball is tied on a string 30 cm long, and is oscillating like a pendulum.


See figure below. It is initially pulled back at 90◦ angle with the vertical, and is
released with a push so that the ball reaches its maximum height back and forth.
If it reaches its maximum height again after 3 seconds, find its height 10 seconds
after it was released.

Hint. The height h(t) (in cm) of the ball at time t (in seconds) is given by
𝜋 3
h(t) = |30sin 3 (𝑡 − 2)| + 12.

2. A variable star is a star whose brightness fluctuates as observed from Earth. The
magnitude of visual brightness of one variable star ranges from 2.0 to 10.1, and
it takes 332 days to observe one maximum brightness to the next. Assuming that
the visual brightness of the star can be modeled by the equation y = asinb(t−c) +
d, t in days, and putting t = 0 at a time when the star is at its maximum brightness,
find the constants a, b, c and d, where a, b > 0 and c the least nonnegative number
possible.

103
Name: _______________________________________________ Date: _____________
Grade/Section: _____________________________ Score: ____________

Title of the Activity: The Use of Identity to Find it”s Value


Most Essential Learning Competency: Determine whether an equation is an identity
or a conditional equation and apply trigonometric identities to find other trigonometric
values.
K to 12 BEC CG: STEM_PC11T-IIe-1 and STEM_PC11T-IIe-g-1

Directions: Answer the following activities accurately.

I. Find the domain of the equation.


3
1. √𝑥 + 2 − √𝑥 = 2𝑥

2. 𝑠𝑖𝑛3 𝑥 = sin 𝑥 + 1

3. tan 𝑥 + cot 𝑥 = sin 𝑥

𝑥+1
4. + cos 𝑥 = csc 𝑥
𝑥 2 −1

5. 2𝑡𝑎𝑛2 𝑥 = 2 cot 𝑥 + 1

II. Given the initial information, use the identities to find the values of the trigonometric
functions of 𝜃.
5
1. csc 𝜃 = 3 𝑎𝑛𝑑 tan 𝜃 > 0

12
2. tan 𝜃 = 𝑎𝑛𝑑 cos 𝜃 < 0
5

104
3 3𝜋
3. csc 𝜃 = 2 𝑎𝑛𝑑 π < 𝑥 > 2

7 3𝜋
4. cot 𝜃 = 5 𝑎𝑛𝑑 < 𝜃 < 2𝜋
2

1+tan 𝑥
5.
1+cot 𝑥

105
Name: _______________________________________________ Date: _____________
Grade/Section: _____________________________ Score: ____________

Title of the Activity: The Use of Identity to Find it”s Value


Most Essential Learning Competency: Determine whether an equation is an identity
or a conditional equation and apply trigonometric identities to find other trigonometric
values.
K to 12 BEC CG: STEM_PC11T-IIe-1 and STEM_PC11T-IIe-g-1

Directions: Answer the following activities accurately.

I. Determine whether the given equation is an identity or a conditional equation. If it


is an identity, prove it; otherwise, provide a value of the variable in the domain that
does not satisfy the equation.

1. sin 𝑥 cos 𝑥 = 1

2. sin2 𝑥 = cos 𝑥 − 1

3. (sin 𝑥 − 𝑐0𝑠 𝑥 )2 + (sin 𝑥 + 𝑐0𝑠 𝑥 )2 = 2

4. tan(−𝑥 ) cot 𝑥 = −1

5. 2 − 𝑠𝑖𝑛2 𝑥 = sec 𝑥 + cos 𝑥

106
II. Find the exact value of each expression.

1. cos 255°

𝜋
2. tan 12

3. sin 735°

4. cot 285°

𝜋 2𝜋 𝜋 𝜋
5. cos ( ) cos ( ) − sin ( ) sin (2 )
9 9 9 9

III. Simplify the following expression.


1. cos (𝜋 − 𝑥)

2. tan (𝑥 + 𝜋)

3𝜋
3. sin ( + 𝑥)
2

4. cos(𝑥 − 𝜋)

107
Name: _______________________________________________ Date: _____________
Grade/Section: _____________________________ Score: ____________

Title of the Activity: The Use of Identity to Find it”s Value


Most Essential Learning Competency: Determine whether an equation is an identity
or a conditional equation and apply trigonometric identities to find other trigonometric
values.
K to 12 BEC CG: STEM_PC11T-IIe-1 and STEM_PC11T-IIe-g-1

Directions: Answer the following activities accurately.

I. Given some information about a and b, find sin (a+b), cos (a-b), and tan (a+b).

3 12
1. sin 𝑎 = 5 , cos 𝑏 = 13
, 𝑎 𝑙𝑖𝑒𝑠 𝑖𝑛 𝑡ℎ𝑒 𝑡ℎ𝑖𝑟𝑑 𝑞𝑢𝑎𝑑𝑟𝑎𝑛𝑡, 𝑎𝑛𝑑 𝑏 𝑖𝑛 𝑡ℎ𝑒 𝑓𝑖𝑟𝑠𝑡 𝑞𝑢𝑎𝑑𝑟𝑎𝑛𝑡.

1 3 𝜋 𝜋
2. cos 𝑎 = 2 , tan 𝑏 = − 2 , < 0 < 𝑎 < 2 , 𝑎𝑛𝑑 <𝑏<𝜋
2

5 3
3. sec 𝑎 = − 2 , cot 𝑏 = 5 , 𝑎 𝑖𝑛 𝑄𝐼𝐼, 𝑎𝑛𝑑 𝑏 𝑖𝑛 𝑄𝐼𝐼𝐼

108
II. Given some information about , find sin 2, cos 2, and tan 2.

1 𝜋
1. cos 𝜃 = − 4 𝑎𝑛𝑑 2
<𝜃<𝜋

5
2. sec 𝜃 = 2 𝑎𝑛𝑑 sin 𝜃 > 0

3𝜋
3. tan 𝜃 = −2 𝑎𝑛𝑑 < 𝜃 < 2𝜋
2

3
4. sin 𝜃 = 5 𝑎𝑛𝑑 tan 𝜃 < 0

III. Given some initial information, use the identities to find the values of the
trigonometric functions of .

5
1. csc 𝜃 = 𝑎𝑛𝑑 tan 𝜃 > 0
3

12
2. tan 𝜃 = 𝑎𝑛𝑑 cos 𝜃 < 0
5

3 3𝜋
3. csc 𝜃 = − 2 𝑎𝑛𝑑 𝜋 < 𝑥 < 2

7 3𝜋
4. cot 𝜃 = − 5 𝑎𝑛𝑑 < 𝜃 < 2𝜋
2

5. sin 𝜃 = −1

109
Name: _______________________________________________ Date: _____________
Grade/Section: _____________________________ Score: ____________

Title of the Activity: The Use of Identity to Find it”s Value


Most Essential Learning Competency: Determine whether an equation is an identity
or a conditional equation and apply trigonometric identities to find other trigonometric
values.
K to 12 BEC CG: STEM_PC11T-IIe-1 and STEM_PC11T-IIe-g-1

Directions: Answer the following activities accurately.

𝜃 𝜃 𝜃
I. Given some information about θ, where 0  2, find sin , cos 𝑎𝑛𝑑 tan .
2 2 2
1 𝜋
3. cos 𝜃 = − 𝑎𝑛𝑑 <𝜃<𝜋
4 2

5
4. sec 𝜃 = 𝑎𝑛𝑑 sin 𝜃 > 0
2

3𝜋
5. tan 𝜃 = −2 𝑎𝑛𝑑 < 𝜃 < 2𝜋
2

3
6. sin 𝜃 = 𝑎𝑛𝑑 tan 𝜃 < 0
5

II. Express each expression as trigonometric expression, but do not find the exact value.

1. 2 sin 10° cos 10°

7𝜋
1−cos( )
2. √ 6
2

110
3𝜋
3. 1 − 2 𝑠𝑖𝑛2 ( 10 )

1+cos 8
4. 2

III. Solve the following.

1−𝑠𝑒𝑐 2 𝑥
1. Express in terms of sin x.
𝑠𝑒𝑐 2 𝑥

2. Express tan 𝑥 sec 𝑥 in terms of cos x.

3. Express all other five trigonometric functions in terms of tan x (allowing  in the
expression).

4. If sec 𝜃 − tan 𝜃 = 3, what is sec 𝜃 + tan 𝜃 ?

111
Name: _____________________________________________ Date: _____________
Grade/Section: ______________________________ Score: ____________

Title of the Activity: The Use of Identity to Solve It’s Problem


Most Essential Learning Competencies: Solve situational problems involving
trigonometric identities.
K to 12 BEC CG: STEM_PC11T-IIg-2

Directions: Solve the following.

1. In an alternating current circuit, the instantaneous power 𝑃(𝑡) at time 𝑡 is given by.
𝑃 (𝑡) = 𝐼𝑚 𝑉𝑚 𝑐𝑜𝑠𝜑𝑠𝑖𝑛2 (𝜔𝑡) − 𝐼𝑚 𝑉𝑚 𝑠𝑖𝑛𝜑𝑠𝑖𝑛(𝜔𝑡)cos(𝜔𝑡),

Where 𝐼𝑚 and 𝑉𝑚 are the maximum current (in amperes) and voltage (in volts),
respectively. Express this function as a product of two sine functions.

2. A particle is moving according to the equation of motion


𝜋 𝜋
𝑠(𝑡) = sin (4𝑡 + ) + sin (4𝑡 + )
3 6
Where 𝑠(𝑡) centimeters is the directed distance of the particle from the origin at 𝑡 seconds.
(i) Express 𝑠(𝑡) in the form 𝑠(𝑡) = 𝑎 sin(𝑏𝑡 + 𝑐).

(ii) Find the amplitude and frequency of the motion. (Here, frequency is defined as the
reciprocal of the period.)

112
Name: _____________________________________________ Date: _____________
Grade/Section: ______________________________ Score: ____________

Title of the Activity: The Use of Identity to Solve It’s Problem


Most Essential Learning Competencies: Solve situational problems involving
trigonometric identities.
K to 12 BEC CG: STEM_PC11T-IIg-2

Directions: Solve the following.

1. Two atmospheric waves in space produce pressure of 𝐹(𝑡) and 𝐺(𝑡) pascals at 𝑡
3𝜋
seconds, where 𝐹 (𝑡) = 0.04cos(2𝜋𝑡) and 𝐺 (𝑡) = 0.04cos(2𝜋𝑡 − ).
4

Express the total pressure 𝑃(𝑡) = 𝐹 (𝑡) + 𝐺(𝑡) in the form 𝑃 (𝑡) = 𝑎 cos(𝑏𝑡 + 𝑐 ).

2. The range 𝑅 of a project fired at an acute angle 𝜃 with the horizontal and with an initial
𝑣2
velocity of 𝑣 meters per second is given by 𝑅 = sin(2𝜃), where g is the acceleration
𝑔

due to gravity, which is 9.81 𝑚/𝑠𝑒𝑐 2 near the Earth’s surface.


(a) An archer targets an object 100 meters away from her position. If she positions her
arrow at an angle 32° and releases the arrow at the speed of 30𝑚/𝑠𝑒𝑐 2 , will she
hit her target?

2
(b) If 𝑠𝑖𝑛𝜃 = 5, solve for 𝑣 when 𝑅 = 50.

(c) Given 𝑣, find the value of 𝜃 that gives the largest possible range. At this 𝜃, what is
the range?

113
3. The figure shows a laser scanner projection system. The optical angle 𝜃, throw distance
𝐷, and projected image width 𝑊 are related by the equation
𝑊
𝐷= 2
𝑐𝑠𝑐𝜃 − 𝑐𝑜𝑡𝜃
𝜃
Solve for W in terms D and 2 .

width of the projected image


throw distance
scanner
optical angle

114
Name: _____________________________________________ Date: _____________
Grade/Section: ______________________________ Score: ____________

Title of the Activity: The Use of Identity to Solve It’s Problem


Most Essential Learning Competencies: Solve situational problems involving
trigonometric identities.
K to 12 BEC CG: STEM_PC11T-IIg-2

Directions: Solve the following.

1. The force F (in pounds) on the back of a person when he or she bends over at an acute angle
0.6𝑊 𝑠𝑖𝑛(𝜃+90)
θ (in degrees) is given 𝑏𝑦 𝐹 = , where W is the weight (in pounds) of the person.
sin 12

(a) Simplify the formula for F.

(b) Find the force on the back of a person whose weight is 154.32 lbs if he bends an angle
of 40∘ .

(c) How many pounds should a person weigh for his back to endure a force of 275 lbs if
he bends 38∘ ?

2. The slope of a mountain makes an angle of 45° with the horizontal. At the base of the
mountain, a cannon is fired at an angle 𝜃 with the horizontal, where 45° < 𝜃 < 90° , and with
initial velocity of 𝑣 m/sec. Neglecting air resistance, the distance 𝑅 (in meters) it drops on the
2√2𝑣 2
slope of the mountain from the base is given by 𝑅 = (𝑠𝑖𝑛𝜃 − 𝑐𝑜𝑠𝜃 )𝑐𝑜𝑠𝜃, where 𝑔 is the
𝑔

acceleration due to gravity in 𝑚/𝑠𝑒𝑐 2 . Express this formula for 𝑅 in terms of 2𝜃.

115
Name: _____________________________________________ Date: _____________
Grade/Section: ______________________________ Score: ____________

Title of the Activity: The Use of Identity to Solve It’s Problem


Most Essential Learning Competencies: Solve situational problems involving
trigonometric identities.
K to 12 BEC CG: STEM_PC11T-IIg-2

Directions: Solve the following.

1. The dual tone multi-frequency is the signal information used in touch-tone phones to
identify which digit you touched on the keypad. It works by adding a pair of sounds,
one with a lower frequency and one with a higher frequency. Refer to the chart below.
For example, the sound created by touching 6 is produced by adding a 770-hertz sound.
(Note that “hertz” is a unit of frequency and is equal to 1 cycle per second.) This sound
is modeled by the equation 𝑠(𝑡) = sin(2𝜋 ∙ 770𝑡) + sin(2𝜋 ∙ 1477𝑡), where 𝑡 is time
in seconds.

(a) Write the equation of the sound created by touching the *(asterisk) key as a product of
sine and cosine functions.

(b) In (a), what is the maximum value of 𝑠(𝑡)?

2. Two non-vertical lines intersects at the point (−3, 2), and one angle between them
measures 30° . If one line is 2𝑦 = 𝑥 + 7, find the equation of the other line.

116
3. The length 𝑠(𝜃) of the shadow cast by a vertical pole when the angle of the sun with
the horizontal is given by
ℎ sin(90° − 𝜃)
𝑠 (𝜃 ) = ,
𝑠𝑖𝑛𝜃
where ℎ is the height of the pole.

𝜃
𝑠(𝜃)

(a) Express 𝑠(𝜃 ) as a single trigonometric expression.

(b) At what angle 𝜃will give the shortest shadow of the pole? Longest shadow?

117
Name: ________________________________________________ Date: _____________
Grade/Section: ______________________________ Score: ____________

Title of the Activity: Limited Yet Existing Values


Most Essential Learning Competency: Illustrate the domain and range of the inverse
trigonometric functions, evaluate an inverse trigonometric expression and solve
trigonometric equations.
K to 12 BEC CG: STEM_PC11T-IIh-1, STEM_PC11T-IIh-2,
and STEM_PC11T-IIh-i-1

Directions: Answer the following activities accurately.

I. Find the exact value of each expression.


1
1. 𝑠𝑖𝑛−1 (− 2)

2. 𝑐𝑜𝑠 −1 0

3. 𝑡𝑎𝑛−1 √3

4. 𝑐𝑠𝑐 −1 1

5. 𝑠𝑒𝑐 −1 (−2)

II. Simplify each expression.


𝜋
6. 𝑐𝑜𝑠 −1 (cos 3 )
𝜋
7. 𝑐𝑠𝑐 −1 (tan 6 )
5𝜋
8. 𝑡𝑎𝑛−1 (tan )
4
𝜋
9. 𝑠𝑖𝑛−1 (cos (− 4 ))
𝜋
10. 𝑐𝑜𝑠 −1 (csc 3 )

118
Name: ________________________________________________ Date: _____________
Grade/Section: ______________________________ Score: ____________

Title of the Activity: Limited Yet Existing Values


Most Essential Learning Competency: Illustrate the domain and range of the inverse
trigonometric functions, evaluate an inverse trigonometric expression and solve
trigonometric equations.
K to 12 BEC CG: STEM_PC11T-IIh-1, STEM_PC11T-IIh-2,
and STEM_PC11T-IIh-i-1

Directions: Answer the following activities accurately.

I. Simplify each expression.


1 1
1. sin (𝑠𝑖𝑛−1 2 + 𝑐𝑜𝑠 −1 2)

√3
2. cos (𝑡𝑎𝑛−1 √3 + 𝑠𝑖𝑛 (− ))
2

3. tan(2𝑡𝑎𝑛−1 (−1))

4 5
4. cos (𝑡𝑎𝑛−1 3 + 𝑐𝑜𝑠 −1 13)

1 √3
5. sin (2𝑠𝑖𝑛−1 2 − 3𝑡𝑎𝑛−1 )
3

119
II. Solve for t in terms of x.

1. 𝑥 = sin 3𝑡

2. 𝑥 = 2 tan(𝑡 + 1)

1
3. 𝑥 = cos(2𝑡 + 1)
2

3
4. 𝑥 = 2 − sec(1 − 𝑡)
2

1
5. 𝑥 = 2
− cot(2 − 3𝑡)

120
Name: ________________________________________________ Date: _____________
Grade/Section: ______________________________ Score: ____________

Title of the Activity: Limited Yet Existing Values


Most Essential Learning Competency: Illustrate the domain and range of the inverse
trigonometric functions, evaluate an inverse trigonometric expression and solve
trigonometric equations.
K to 12 BEC CG: STEM_PC11T-IIh-1, STEM_PC11T-IIh-2,
and STEM_PC11T-IIh-i-1

Directions: Answer the following activities accurately.

I. Find the exact value of each equation.


1. 2 sin 𝑥 + 1 = 0

2. sin 𝑥 tan 𝑥 = 0

3. tan 𝑥 + 1 = 0

4. cos 3𝑥 = 0

5. tan 4𝑥 − 1 = 0.

6. 𝑠𝑒𝑐 2 𝑥 − 1 = 0

7. 𝑠𝑒𝑐 2 𝑥 + 6 tan 𝑥 + 4 = 0

8. cos 2𝑥 + 3 = 5 cos 𝑥

𝑥
9. 𝑐𝑜𝑠 2 𝑥 + 𝑠𝑖𝑛2 2 = 1

𝑥 𝑥
10. 6𝑠𝑒𝑐 2 2 + 3 = 7𝑡𝑎𝑛 2

121
II. Sketch the graph of each function.

1. 𝑦 = 𝑐𝑜𝑠 −1 ( 𝑥 + 1)

2. 𝑦 = 𝑠𝑖𝑛−1 (𝑥 − 2)

3. 𝑦 = 𝑠𝑖𝑛−1 2𝑥

𝑥
4. 𝑦 = 𝑐𝑜𝑠 −1
2

5. 𝑦 = 2𝑐𝑜𝑠 −1 (𝑥 − 1)

122
Name: ________________________________________________ Date: _____________
Grade/Section: ______________________________ Score: ____________

Title of the Activity: Limited Yet Existing Values


Most Essential Learning Competency: Illustrate the domain and range of the inverse
trigonometric functions, evaluate an inverse trigonometric expression and solve
trigonometric equations.
K to 12 BEC CG: STEM_PC11T-IIh-1, STEM_PC11T-IIh-2,
and STEM_PC11T-IIh-i-1

Directions: Answer the following activities accurately.

I. Find the solutions in the interval [0, 2π)


1. 4𝑠𝑖𝑛2 𝑥 − 1 = 0 9. 𝑡𝑎𝑛2 𝑥 = 1 + sec 𝑥

2. 2𝑐𝑜𝑠 2 𝑥 + 3 cos 𝑥 − 2 = 0 10. tan 𝑥 + √3 = sec 𝑥

3. tan 𝑥 − cot 𝑥 = 0

4. 2 𝑠𝑖𝑛2𝑥 = √3

5. 𝑠𝑒𝑐 2 𝑥 − 4 = 0

6. 2𝑠𝑖𝑛2 𝑥 − 5 𝑠𝑖𝑛𝑥 = 3

7. tan 𝑥 + sec 𝑥 = 0

8. 2𝑠𝑖𝑛 2𝑥 = 3 sin 𝑥

123
II. Find the solutions in the interval [0° , 180° ).

1. sin 𝑥 − cos 𝑥 = 0.

2. cot 4𝑥 − 1 = 0

3. 3𝑐𝑜𝑠2𝑥 − 3 cos 𝑥 = 0

4. 6𝑠𝑒𝑐 2 𝑥 + 3 = 7 tan 𝑥

5. 𝑡𝑎𝑛2 𝑥 + tan 𝑥 = 6.

124
Name: _______________________________________________ Date: _____________
Grade/Section: ______________________________ Score: ____________

Title of the Activity: Problems of the Opposite World


Most Essential Learning Competencies: Solve situational problems involving
inverse trigonometric functions and trigonometric equations.
K to 12 BEC CG: STEM_PC11T-IIi-2

Directions: Answer the following activities accurately.

𝜋
1. Solve for 𝑥 in the equation 𝑠𝑖𝑛−1 (𝑥 2 − 2x) = − 2 .

𝜋
2. Solve for 𝑥 in the equation 𝑡𝑎𝑛−1 (4𝑥 2 + 5x − 2) = − 4 .

2 1
3. If 𝑥 be a real number such thatcos 2𝑥 = 3 and cos 𝑥 − 𝑠𝑖𝑛 𝑥 = 2 , what is

cos 𝑥 + sin 𝑥?

4. Solve for 𝑥∈ [−π,π): 16𝑠𝑖𝑛4 𝑥 +1 = 8 𝑠𝑖𝑛2

125
Name: _______________________________________________ Date: _____________
Grade/Section: ______________________________ Score: ____________

Title of the Activity: Problems of the Opposite World


Most Essential Learning Competencies: Solve situational problems involving
inverse trigonometric functions and trigonometric equations.
K to 12 BEC CG: STEM_PC11T-IIi-2

Directions: Answer the following problems accurately.

1. During a leap year, the number of hours of daylight in a city can be modeled by 𝐷(𝑡) =
12 + 2.4 𝑠𝑖𝑛(0.017𝑡 − 1.377), where 𝑡 is the day of the year (that is, 𝑡 = 1 means
January 1, 𝑡 = 60 is February 29, and so on).

2. Find the smallest positive value of Ɵ that satisfies the equation


𝜋 𝜋 3
sin( 3 + Ɵ) + sin( 3 – Ɵ ) = 8

3. A woman is standing 𝑥 ft from a wall with a billboard nailed on it. The billboard is 5ft
tall, and its base is 6 ft above the eye level of the woman. Find the viewing angle
subtended on the eyes of the woman from the base to the top of the billboard.

126
Name: _______________________________________________ Date: _____________
Grade/Section: ______________________________ Score: ____________

Title of the Activity: Problems of the Opposite World


Most Essential Learning Competencies: Solve situational problems involving
inverse trigonometric functions and trigonometric equations.
K to 12 BEC CG: STEM_PC11T-IIi-2

Directions: Answer the following problems accurately.

1. A weight is suspended from a spring and vibrating vertically according


to the equation (𝑡) = 25.2sin(3.8𝑡 − 2.1) , where 𝑓(𝑡) centimeters is the directed
distance means above its central position.
(a.) Find the times when the weight is at its central position.

(b.) For what values of 𝑡 does the weight reach its farthest point below its central
position?

127
2. After getting a job, a man started saving a percentage of his annual income, which can
be modeled by𝑃 (𝑡) = 2.5 cos(0.156𝑡) + 5.2, where 𝑃(𝑡) is the percentage of his annual
income that he was able to save on year 𝑡 after he got a job.
(a.) What percentage of his annual income did he save on the second year?

(b.) On what year right after getting a job did he save the least?

(c.) On what year right after getting a job did he save the most? When would it happen
again?

(d.) If he got his job at the age of 20, how much will he save on the year of his
retirement (that is, when he is 60)?

128
Name: _______________________________________________ Date: _____________
Grade/Section: ______________________________ Score: ____________

Title of the Activity: Problems of the Opposite World


Most Essential Learning Competencies: Solve situational problems involving
inverse trigonometric functions and trigonometric equations.
K to 12 BEC CG: STEM_PC11T-IIi-2

Directions: Answer the following problems accurately.

1. The finance department of a car company conducted a study of their weekly sales in the
past years, and came out with the following approximating function: 𝑠 (𝑡) =
12.18 cos(0.88𝑡 − 7.25) + 20.40, 𝑡 ≥ 0, where 𝑠(𝑡) represents weekly car sales in
million pesos at week 𝑡 (𝑡 = 0 𝑟𝑒𝑝𝑟𝑒𝑠𝑒𝑛𝑡𝑠 𝑡ℎ𝑒 𝑠𝑡𝑎𝑟𝑡 𝑜𝑓 𝑡ℎ𝑒 𝑠𝑡𝑢𝑑𝑦).
(a.) Find the weekly sales at the start of the study.

(b.) Find the projected maximum and minimum weekly sales of the company.

129
(c.) If the company were able to reach its maximum sales this week, when will the
next projected maximum weekly sales and upcoming projected minimum
weekly sales be?

(d.) After the start of the study, when did the company experience a weekly sales of
only 10 million for the first time?

2. After many years in business, the financial analyst of a shoe company projected that the
monthly costs of producing their products and monthly revenues from the sales of their
products are fluctuating according to the following formulas:
𝐶 (𝑡) = 2.6 + 0.58 sin(0.52𝑡 − 7.25) and 𝑅(𝑡) = 2.6 + 1.82cos(0.52𝑡 − 7.25),
where 𝐶 (𝑡) and 𝑅(𝑡)are the costs and revenues in million of pesos at month 𝑡 (t = 0
represents January 2010). About how many months after January 2010 did the company
experience a zero profit for the first time?

130
Answer Key:
Simple Activities: Round and Round Activity
5𝜋
I. 1. −110° 6. 6

2. −940° 7. −250°
8𝜋 13𝜋
3. 8. −
3 6
17𝜋 55𝜋
4. 9.
4 12

5. 350° 10. 370°

II.
No. Degree Measure Radian Measure
1. 330° 11𝜋
6
2. 300° 5𝜋
3
3. 315° 7𝜋
4
4. 270° 3𝜋
2
5. 225° 5𝜋
4
6. 240° 4𝜋
3
7. 210° 7𝜋
6
8. 180° 𝜋
9. 150° 5𝜋
6
10. 135° 3𝜋
4
11. 120° 2𝜋
3
12. 90° 𝜋
2
13. 60° 𝜋
3
14. 45° 𝜋
4
15. 30° 𝜋
6
16. 0° or 360° 0 or 2𝜋

131
4𝜋 7𝜋 25𝜋 17𝜋
III. A. 1. 2. 3. 4. −
3 4 36 9

B. 1. 855° 2. 45° 3. −430° 4. −310°


Moderate Activities: Draw and Compute Me Logically!
I.

1. 6.

2. 7.

3. 8.

4. 9.

5. 10.

132
II. 1. 128° 46’ 12” 5. 63° 45’
2. 232. 1325° 6. 200° 19’ 30”
3. −154.795° 7. −317° 3’ 36”
4. −0° 55’ 21” 8. 179° 59’ 56”

III. A. 1. III 2. III


𝜋 𝜋 𝜋
B. 1. 12 2. 4 3. 4 4. 10° 5. 70° 6.

40°
𝜋 5𝜋 7𝜋 11𝜋 13𝜋
C. 1. 6 , , , , 2. One: 0° = 0 radians
6 6 6 6

Challenging Activities: App-App Activity


I.
1. 330° is a Quadrant IV angle; coterminal with 600° and −30°.

2. −135° is a Quadrant III angle; coterminal with 225° and −495°.

5𝜋 17𝜋 7𝜋
3. is a Quadrant II angle; coterminal with and − .
6 6 6

11𝜋 𝜋 5𝜋
4. − is a Quadrant I angle; coterminal with 3 and − .
3 3

133
𝜋 3𝜋 5𝜋
5. − 2 lies on the negative y-axis; coterminal with and − .
2 2

6. 3𝜋 lies on the negative x-axis; coterminal with 𝜋 and – 𝜋.

II.
1. About 30.12 miles per hour
2. About 6,274.52 revolutions per minute
3. About 3.33 miles per hour
4. About 53.55 miles per hour

Enrichment Activities: Wrapped Me in Round Activity!


1. 70 miles per hour
2. About 4.32 miles per hour

Answer Key:
Simple Activities: Tracking The Starting Point
I. 1. 22° 10. 30°

2. 48° √3
II. 1. − 2
3. 63°
2. −√2
4. 56° 1
3. 2
5. 60° 1
4. − 2
6. 45°
5. −2
7. 60° 1
6. 2
8. 25° 7. 2
9. 20° 8. −1

134
1 10. −1
9. 2

Moderate Activities: Tracking The Starting Point


2√3 1
I. 1. 6. −
3 2

√3
2. 2 7. 2

2√3
3. −√3 8. 3

√3 2√3
4. 9.
3 3

√3
5. −√3 10. − 3

II. 1. 1
2. √3

3. 0

4. 0

5. 0

Challenging Activities: Tracking The Starting Point

4𝜋 √3 3𝜋 √2
I. 1. 1 II. 1. sin − = 2. sin =
3 2 4 2

4𝜋 1 3𝜋 √2
2. 0 cos − = −2 cos =−
3 4 2
1 4𝜋 3𝜋
3. − 2 tan − 3
= −√3 tan 4
= −1

√3 4𝜋 2√3 3𝜋
4. − csc − = csc = √2
3 3 2 4
4𝜋 3𝜋
5. 1 sec − = −2 sec = −√2
3 4

4𝜋 √3 3𝜋
sin − =− cot = −1
3 3 4

135
√3 2√3 √3
III. 1. sec 𝜃 = 2 , sin 𝜃 = − , tan 𝜃 = − √3 , csc 𝜃 = − , cot 𝜃 = −
2 3 3
17 15 8 17 15
2. csc 𝜃 = , cos 𝜃 = 17 , tan 𝜃 = 15 , sec 𝜃 = 15 , cot 𝜃 =
8 8

√13 3√13 3 √13 2


3. sec 𝜃 = , sin 𝜃 = − , tan 𝜃 = − , csc 𝜃 = − , cot 𝜃 = −
2 13 2 3 3

Enrichment Activities: Tracking The Starting Point


3 4 3 5 5 4
I. 1. sin 𝜃 = 5 , cos 𝜃 = − 5 , tan 𝜃 = − 4 , csc 𝜃 = 3 , sec 𝜃 = − 4 , cot 𝜃 = − 3
12 5 12 13 13 5
2. sin 𝜃 = − 13 , cos 𝜃 = − 13 , tan 𝜃 = , csc 𝜃 = − 12 , sec 𝜃 = − , cot 𝜃 = 12
5 5
24 7 24 25 25 7
3. sin 𝜃 = 25 , cos 𝜃 = 25 , tan 𝜃 = , csc 𝜃 = 24 , sec 𝜃 = , cot 𝜃 = 24
7 7

4√3 1 7√3 √3
4. sin 𝜃 = − , cos 𝜃 = 7 , tan 𝜃 = −4√3 , csc 𝜃 = − , sec 𝜃 = 7 , cot 𝜃 = − 12
7 12

√91 3 √91 10√91 10 3√91


5.sin 𝜃 = − , cos 𝜃 = − 10 , tan 𝜃 = , csc 𝜃 = − , sec 𝜃 = − , cot 𝜃 =
10 3 91 4 91

√530 23√530 1 √530


6.sin 𝜃 = ,cos 𝜃 =− ,tan 𝜃 = − 23,csc 𝜃 = √530 ,sec 𝜃 = − ,cot 𝜃 = −23
530 530 23

2√5 √5 √5 1
7. sin 𝜃 = − , cos 𝜃 = , tan 𝜃 = −2 , csc 𝜃 = − , sec 𝜃 = √5 , cot 𝜃 = − 2
5 5 2

√15 1 4√15 √15


8. sin 𝜃 = , cos 𝜃 = − 4 , tan 𝜃 = −√15 , csc 𝜃 = , sec 𝜃 = −4 , cot 𝜃 = −
4 15 15

√6 √30 √5 √30
9. sin 𝜃 = − , cos 𝜃 = − , tan 𝜃 = , csc 𝜃 = −√6 , sec 𝜃 = − , cot 𝜃 = √5
6 6 5 5

2√2 1 3√2 √2
10. sin 𝜃 = , cos 𝜃 = 3 , tan 𝜃 = 2√2 , csc 𝜃 = , sec 𝜃 = 3 , cot 𝜃 =
3 4 4

√3 1 1 √3
II. 1. 𝑃(𝜃 ) = (− , − 2) 2. 𝑃(𝜃 ) = (− 2 , ,)
2 2

19𝜋 1 32𝜋 √3
sin =− sin =
6 2 3 2

19𝜋 √3 32𝜋 1
cos =− cos = −2
6 2 3

19𝜋 √3 32𝜋
tan = tan = −√3
6 3 3

19𝜋 32𝜋 2√3


csc = −2 csc =
6 3 3

19𝜋 2√3 32𝜋


sec =− sec = −2
6 3 3

19𝜋 32𝜋 √3
cot = √3 cot =−
6 3 3

136
Answer Key:
Simple Activities: Draw Me Meticulously
I. 1. amplitude = 4 , period = 2π , phase shift = 0 , domain = ℝ , range = [-4 , 4]

2. amplitude = 3 , period = 2π , phase shift = 0 , domain = ℝ , range = [-3, 3]

3. amplitude = 1 , period = 8π , phase shift = 0 , domain = ℝ , range = [-1 , 1]

4. amplitude = 1 , period = π , phase shift = 0 , domain = ℝ , range = [-1 , 1]


𝜋
5. amplitude = 1 , period = 2 , phase shift = 0 , domain = ℝ , range = [1 , 3]

6. amplitude = 1 , period = 2π , phase shift = 0 , domain = ℝ , range = [-2 , 0]


1 2𝜋 1 1
7. amplitude = 2 , period = , phase shift = 0 , domain = ℝ , range = [− 2, 2]
3

8. amplitude = 3 , period = 2π , phase shift = 0 , domain = ℝ , range = [-3 , 3]

9. amplitude = 2 , period = 4π , phase shift = 0 , domain = ℝ , range = [1 , 5]


𝜋
10. amplitude = 1 , period = 2π , phase shift = , domain = ℝ , range = [-1 , 1]
4

𝜋
II. 1. 𝑦 = 6𝑠𝑖𝑛 2 𝑡
𝜋
2. 𝑦 = 5𝑐𝑜𝑠 5 𝑡
2𝜋 5
3. 𝑦 = 9𝑐𝑜𝑠 (𝑡 − 2)
5

Moderate Activities: Draw Me Meticulously

1. period = 2π , phase shift = 0 , domain = {𝑥|𝑥 ≠ 𝑘𝜋, 𝑘 ∈ ℤ} , range = ℝ


2. period = π , phase shift = 0 , domain = {𝑥|𝑥 ≠ 𝑘𝜋, 𝑘 ∈ ℤ} , range = ℝ
𝜋
3. period = π , phase shift = 0 , domain = {𝑥|𝑥 ≠ (2𝑘 + 1) , 𝑘 ∈ ℤ} , range = ℝ
2
𝜋
4. period = 2π , phase shift = 0 , domain = {𝑥|𝑥 ≠ (2𝑘 + 1) 2 , 𝑘 ∈ ℤ} ,
range = (−∞, −1] ∪ [1, ∞)

2𝜋 𝜋
5. period = , phase shift = 0 , domain = {𝑥|𝑥 ≠ (2𝑘 + 1) 6 , 𝑘 ∈ ℤ} ,
3
range = (−∞, −1] ∪ [1, ∞)
6. period = 2π , phase shift = 0 , domain = {𝑥|𝑥 ≠ 𝑘𝜋, 𝑘 ∈ ℤ} , range = (−∞, −3] ∪
[3, ∞)

137
3𝜋
7. period = 3𝜋, phase shift = 0 , domain = {𝑥|𝑥 ≠ (2𝑘 + 1) , 𝑘 ∈ ℤ} ,
6
range = (−∞, −4] ∪ [4, ∞)
𝜋
8. period = 𝜋, phase shift = −𝜋 , domain = {𝑥|𝑥 ≠ (2𝑘 + 1) 2 , 𝑘 ∈ ℤ} ,
range = ℝ
𝜋
9. period = 𝜋 , phase shift = , domain = {𝑥 |𝑥 ≠ 𝑘𝜋, 𝑘 ∈ ℤ} , range = ℝ
2
𝜋 𝜋
10. period = 𝜋, phase shift = 4 , domain = {𝑥|𝑥 ≠ (2𝑘 − 1) 4 , 𝑘 𝑒𝑣𝑒𝑛 𝑖𝑛𝑡𝑒𝑔𝑒𝑟} , range =

II.1.

2.

Challenging Activities: Draw Me Meticulously


5𝜋
I. 1. Starting with cos θ, vertically stretch by 5, translate left 6
1 3𝜋
2. Starting with sin θ, horizontally shrink by 8, translate left 32 , translate up 5
𝜋
3. Starting with sin θ, translate right 36, translate down 2
1 𝜋
4. Starting with tan θ, vertically shrink by , translate right
10 3

138
5𝜋
II. 1. Amplitude:  , Period: 2𝜋, Phase shift: Right 6
, Vert. shift: None , Min: −3 , Max: 3

2. Amplitude:  , Period: 2𝜋, Phase shift: None , Vert. shift: None , Min: −3 , Max: 3

Enrichment Activities: Draw Me Meticulously


𝜋 11𝜋 7𝜋 2𝜋
I. 1. Period: 2 , Phase shift: Right , Vert. shift: Down, Vert. Asymptotes: 𝑥 = ,𝑥 =
12 6 3

139
5𝜋
2. Period:3𝜋, Phase shift: Left , Vert. shift: None, Vert. Asymptotes: 𝑥 = −𝜋 , 𝑥 = −4𝜋
2

3𝜋 3𝜋 𝜋
3. Period: 4𝜋, Phase shift: Left 2
, Vert. shift: None, Vert. Asymptotes: 𝑥 = − 2
,𝑥 = 2

3𝜋 5𝜋 𝜋
4. Period: 4𝜋, Phase shift: Right , Vert. shift: None, Vert. Asymptotes: 𝑥 = ,𝑥 =
2 2 2

140
II. 1. Amplitude: 2 , Period: 2𝜋 𝜋
, Phase shift: Left , Vert. shift: Up 2 , Min: 0 , Max: 4
3 6

2. Amplitude: 1 , Period: 8𝜋, Phase shift: Left π , Vert. shift: Down 2 , Min: −3 , Max: −1

Answer Key:
Simple Activity: Problems of What Goes Around

I. 1.any integer
2. any even integer
𝜋
3.
4
𝜋
4. 2 +
3
5. −2 < a < 2

141
Moderate Activity: Problems of What Goes Around
1
I. 1. y = 3sin20π(t− )−2
40

2. about the point ( -1.87, 5. 11)

Challenging Activity: Problems of What Goes Around

𝜋 𝜋 7
I. 1. y = 6cos 7 t or y = 6sin (t + 2), location of the weight after 20 seconds: about 5.4 cm
7
below the resting position
𝜋 15
2. y = 15cos 15(t− )+ 16.5
2

Enrichment Activity: Problems of What Goes Around


I. 1. 27 cm
2. a =4.05
π
b =166

c =249
d= 6.05

Answer Key:
Simple Activities: The Use of Identity to Find it”s Value
I. 1.{𝑥 |𝑥 ≥ 0} II. 1. 𝑐𝑜𝑠 2 𝜃

2. ℝ 2. 1

𝑘𝜋
3. − sin 𝑥
3. ℝ ∖ { 2 |𝑘 𝜖 ℤ}
4. 1
4. ℝ ∖ {{−1,1} ∪ {𝑘𝜋|𝑘 ∈ ℤ}} 5. tan 𝑥

𝑘𝜋
5. ℝ ∖ { 2 |𝑘 𝜖 ℤ}

Moderate Activities: The Use of Identity to Find it”s Value


I. 1. Conditional equation, 𝑥 = 2
𝜋
2. Conditional equation, 𝑥 = 2

142
3. Identity
𝜋
4. Conditional equation, 𝑥 = 2

𝜋
5. Conditional equation, 𝑥 = 2

II. 1.
√2−√6 III. 1. − cos 𝑥
4
2. tan 𝑥
2. 2 − √3
3. − cos 𝑥
√6−√2
3. 4
4. − cos 𝑥
4. −2 + √3
1
5. 2

Challenging Activities: The Use of Identity to Find it”s Value


56 63 56
I. 1. sin(𝑎 + 𝑏) = − ; cos(𝑎 − 𝑏) = − ; tan(𝑎 + 𝑏) = −
65 65 33

3√13−2√39 3√39−2√13 24−13√3


2. sin(𝑎 + 𝑏) = ; cos(𝑎 − 𝑏) = ; tan(𝑎 + 𝑏) =
26 26 23

10√34−3√714 6√34−5√714 −375+68√28


3. sin(𝑎 + 𝑏) = ; cos(𝑎 − 𝑏) = ; tan(𝑎 + 𝑏) =
170 170 489

√15 7 √15
II. 1. sin 2𝜃 = ; cos 2𝜃 = − 8; tan 2𝜃 = −
8 7

4√21 17 4√21
2. sin 2𝜃 = ; cos 2𝜃 = − ; tan 2𝜃 = −
25 25 17
4 3 4
3. sin 2𝜃 = − ; cos 2𝜃 = − ; tan 2𝜃 = −
5 5 3
24 7 24
4. sin 2𝜃 = − 25; cos 2𝜃 = 25; tan 2𝜃 = − 7
3 4 5 3 4
III. 1. 𝜃in QI; sin 𝜃 = , cos 𝜃 = √1 − 𝑠𝑖𝑛2 𝜃 = , sec 𝜃 = , tan 𝜃 = , cot 𝜃 =
5 5 4 4 3
5 13 5
2. 𝜃 in QII; cot 𝜃 = − , sec 𝜃 = √𝑡𝑎𝑛2 𝜃 + 1 = − , cos 𝜃 = , sin 𝜃 =
12 15 13
13
√1 − 𝑐𝑜𝑠 2 𝜃, csc 𝜃 = 12

2 √5 3√5 2√5 √5
3. θ in QIII; sinθ = − 3 , cosθ = −√1 − 𝑠𝑖𝑛2 θ = − 3 , secθ = − , tanθ = , cotθ =
5 5 2

5 √74 7√74 √74


4.θ in QIV; tanθ = − 7 , secθ = √𝑡𝑎𝑛2 θ + 1 = , cosθ = , cscθ = −√𝑐𝑜𝑡 2 θ + 1 = -
7 74 5
5√74
, sinθ = −
74

5. θ coterminal with ; cscθ = −1, cosθ = 0, secθ undefined,tan θ undefined, cotθ = 0
2

143
Enrichment Activities: The Use of Identity to Find it”s Value
𝜃 √10 𝜃 √6 𝜃 √15
I. 1.sin 2 = 4
, cos 2 = − 4
, tan 2 = 3

𝜃 √30 𝜃 √70 𝜃 √21


2. sin 2 = 10
, cos 2 = − 10
, tan 2 = 7

𝜃 √50−10√5 𝜃 √50+10√5 𝜃 1−√5


3. sin 2 = , cos 2 = − , tan 2 =
10 4 3

𝜃 3√10 𝜃 √10 𝜃
4. sin 2 = , cos 2 = − , tan 2 = 3
10 10

II. 1.sin 20°


7𝜋
2. sin 12
3𝜋
3. cos 5

4. cos 2 4

III. 1. − sin2 𝑥
√1−cos2 𝑥
2. ± cos2 𝑥
tan 𝑥 1 1
3.sin 𝑥 = ; cos 𝑥 = ; cot 𝑥 = tan 𝑥 ; sec 𝑥 = ±
±√𝑡𝑎𝑛2 𝑥+1 ±√𝑡𝑎𝑛2 𝑥+1
±√𝑡𝑎𝑛2 𝑥+1
√𝑡𝑎𝑛2 𝑥 + 1; csc 𝑥 = tan 𝑥
1
4. 3

Answer Key:
Simple Activity: The Use of Identity to Solve It’s Problem
1. 𝑃 (𝑡) = 𝐼𝑚 𝑉𝑚 𝑠𝑖𝑛(𝜔𝑡)[𝑐𝑜𝑠𝜑 sin(𝜔𝑡) − 𝑠𝑖𝑛𝜑cos(𝜔𝑡)]
= 𝐼𝑚 𝑉𝑚 𝑠𝑖𝑛(𝜔𝑡)sin(𝜑 − 𝜔𝑡)
√2+√6 𝜋
2. (i) 𝑠(𝑡) = 𝑠𝑖𝑛 (4𝑡 + 4)
6

√2+√6 4 2
(ii) Amplitude = ; frequency = 2𝜋 = 𝜋
6

144
Moderate Activity: The Use of Identity to Solve It’s Problem
3𝜋
1. 𝑃 (𝑡) = 0.04√2 − √2 cos (2𝜋𝑡 − )
8
302
2. (𝑎)𝑅 = 9.81 ∙ sin(2 ∙ 32° ) ≈ 82.46 < 100 Answer: NO

(𝑏)𝑣 ≈ 25.86 𝑚/𝑠𝑒𝑐


𝑣2
(𝑐 )𝜃 = 45° , largest 𝑅 = *To reach the largest 𝑅, sin(2𝜃) must be 1.
𝑔

𝜃
3. 𝑊 = 2𝐷𝑡𝑎𝑛 2

Challenging Activity: The Use of Identity to Solve It’s Problem

1. (a) 𝐹 (𝜃 ) = 0.6𝑊 𝑐𝑜𝑠𝜃


𝑠𝑖𝑛 12°

(𝑏) 𝐹 ≈ 340.46 𝑙𝑏𝑠


(𝑐)𝑊 ≈ 121.17𝑙𝑏𝑠
√2𝑣 2
2. 𝑅 = (𝑠𝑖𝑛 2𝜃 − 𝑐𝑜𝑠 2𝜃 − 1)
𝑔

Enrichment Activity: The Use of Identity to Solve It’s Problem

1. (a) 𝑠(𝑡) = 2 sin(2150𝜋𝑡) cos(536𝜋𝑡)


(b) Max value = 2 occurring at 𝑡 = 0.75 + 𝑘, 𝑘 nonnegative integer\
5√3+8
2. 𝑦 = (𝑥 + 3) + 2
11

3. (a) 𝑠(𝜃 ) = ℎ 𝑐𝑜𝑡𝜃; (b) Shortest shadow occurs at 𝜃 = 90° . But the length of the
shadow increases when the value of 𝜃 approaches 0° ; that is, no maximum length for
the shadow.
Answer Key:
Simple Activities: Limited Yet Existing Values

I. II.
1. 330° 1. 60°
2. 90° 2. Undefined
3. 60° 3. 45°
4. 90° 4. 45°
5. 240° 5. 30°

145
Moderate Activities: Limited Yet Existing Values
I. II.
1. 1 1
1. 𝑡 = 𝑠𝑖𝑛−1
3
2. 1
3. Undefined 𝑥
2. 𝑡 = 𝑡𝑎𝑛−1 2 − 1
33
4. − 65
1
5. − 2 𝑐𝑜𝑠−1 (2𝑥)−1
3. 𝑡 = 2

2
4. 𝑡 = 1 − 𝑠𝑒𝑐 −1 ( (2 − 𝑥 ))
3

2 1 1
5. 𝑡 = 3
− 3
𝑐𝑜𝑡 −1 (2 − 𝑥)

Challenging Activities: Limited Yet Existing Values


I.
1. 210°, 330° 6. 0°, 180°
2. 0°, 180° 7. 135°, 315°, 101.31°, 281.31°
3. 135°, 315° 8. 60°, 300°
4. 90°,270° 9. 0°, 120°, 240°, 360°
5. 11.25°, 56.25° 10. No Solution

II.
1. 2.

146
3. 4.

5.

Enrichment Activities: Limited Yet Existing Values


I.
𝜋 5𝜋 7𝜋 11𝜋 7𝜋 11𝜋
1. , , , 6. ,
6 6 6 6 6 6
𝜋 5𝜋 3𝜋
2. 3 3
, 7. 2
𝜋 5𝜋 3 3
3. , 8. 0, 𝜋, 𝑐𝑜𝑠 −1 4 , 2𝜋 − 𝑐𝑜𝑠 −1 4
4 4
𝜋 𝜋 7𝜋 4𝜋 𝜋 5𝜋
4. , , , 9. 0, ,
6 3 6 3 3 3
𝜋 2𝜋 4𝜋 5𝜋 11𝜋
5. , , , 10. 6
3 3 3 3

II.
1. 45°
2. 11.25° , 56.25° , 101.25° , 146.25°
3. 0° , 120°
4. 161.6° , 56.3°
5. 63.4° , 108.4°

147
Answer Key:
Simple Activity: Problems of the Opposite World
I. 1. .𝑥= 1
3
2. 𝑥 = -2, 4
4
3. cos 𝑥 + 𝑠𝑖𝑛𝑥 = 3
5𝜋 𝜋
4. 𝑥 = ± ,±6
6

Moderate Activity: Problems of the Opposite World


I. 1. (a.) 173 days, so the day would be June 21
(b.) 287 days, so the day would be October 13
√3 √3
2. 𝜃 = 𝑐𝑜𝑠 −1 ( 8 ) 3. 𝜃 = 𝑐𝑜𝑠 −1 ( 8 )
15𝑥
3. 𝑡𝑎𝑛−1
𝑥2 +126

Challenging Activity: Problems of the Opposite World


I.1. (a.) 𝑡 ≈ 0.55 + 0.83𝑘
(b.) 𝑡 ≈ 1.79 + 1.65𝑘
2. (a.) 7.58% of his annual income
(b.) 20 years after getting a job

(c.) 40 years after getting a job


(d.) 7.7% of his annual income

Enrichment Activities: Problems of the Opposite World


I. 1. (a.) 27.32
(b.) Maximum =32.58 million pesos and Minimum = 8.22 million pesos
(c.) 𝑃 = 7 weeks
(d.) 𝑡 ≈ 11.29 + 7.14𝑘 𝑜𝑟 𝑡 ≈ 5.29 + 7.14𝑘
2. The company experienced zero profit for the first time about 16.37+6.04(-2) ≈ 4.29 or 4
months after January 2010.

148
References:
A. Books
R.N. Aufmann, V.C. Barker, and R.D. Nation. 2008. College Trigonometry,
Houghton Mifflin Company.

E.A. Cabral, M.L.A.N. De Las Peñas, E.P. De Lara-Tuprio, F.F. Francisco, I.J.L.
Garces, R.M. Marcelo, and J.F. Sarmiento. 2010. Precalculus, Ateneo de Manila
University Press.

R. Larson. 2014. Precalculus with Limits, Brooks/Cole, Cengage Learning.

L. Leithold. 2002. College Algebra and Trigonometry, Addison Wesley Longman


Inc., 1989, reprinted by Pearson Education Asia Pte. Ltd.

M.L. Lial, J. Hornsby, and D.I. Schneider, 2001, College Algebra and Trigonometry
and Precalculus, Addison-Wesley Educational Publisher, Inc.

J. Stewart, L. Redlin, and S. Watson. 2012. Precalculus: Mathematics for Calculus,


Brooks/Cole, Cengage Learning.

C. Stitz, J. Zeager. 2013. College Trigonometry.

M. Sullivan. 2012. Algebra & Trigonometry, Pearson Education, Inc.

C. Young, 2013, Algebra and Trigonometry, John Wiley & Sons, Inc.

B. Government Publication

Commission on Higher Education K to 12 Transition Program Management Unit. 2016,


TEACHING GUIDE FOR SENIOR HIGH SCHOOL Precalculus, 4th Floor,
Commission on Higher Education, C.P. Garcia Ave., Diliman, Quezon City.
Government of the Philippines. Department of Education. 2016. Precalculus Teacher’s
Guide. Quezon City

C. Online and Other Resources


GeoGebra. n.d. GeoGebra. https://www.geogebra.org/apps/
https://www.shelovesmath.com/precal/systems-non-linear-equations/

https://www.chilimath.com/lessons/advanced-algebra/systems-non-linear-equations/

http://www.kutasoftware.com/freeica.html

http://www.kutasoftware.com/freeica.html

http://www.columbia.edu/itc/sipa/math/summation.html

149
https://www.purposegames.com/worksheet/radians-quiz#download
https://www.stitz-zeager.com/szprecalculus07042013.pdf

https://www.thatquiz.org/tq/preview?c=bjfc8146&s=o1vs9m

https://cdn.kutasoftware.com/Worksheets/Precal/04%20%20Graphs%20of%20Trig%2
0Functions.pdf

150
151

You might also like